krok 2 - 2009 question paper (stomatology)

29
Krok 2 Stomatology 2009 1 1. A 45-year-old female patient complains about pain caused by thermal srtimuli and spontaneous pain in the 26 tooth. A week ago this tooth was treated for pulpitis. Objectively: the 26 tooth is lled, percussion is painful, thermal sti- mulus causes slowly intensifying long- lasting pain. X-ray picture shows that the palatine canal is lled by 2/3, buccal canals contain no lling material. What is the most probable cause of this complication? A. Incomplete pulp extirpation B. Inadequate canal lling C. Infection D. Inammation in the periodontium E. Trauma of periodontium tissue 2. Preventive examination of a 7-year- old schoolboy revealed unremovable grey-and-white layerings on the mucous membrane of cheek along the line of teeth joining. Mucous membrane is sli- ghtly hyperaemic, painless on palpation. The boy is emotionally unbalanced, bi- tes his cheeks. What is the most likely di- agnosis? A. Mild leukoplakia B. Chronic recurrernt aphthous stomatitis C. Chronic candidous stomatitis D. Lichen ruber planus E. Multiform exudative erythema 3. A 48-year-old patient complains about itching gums. Objectively: gums are dense, of normal colour; cold water causes short-term toothache. X-ray pi- cture shows osteosclerosis of interalveolar septa (ne granular bone texture), hei- ght of interalveolar septa and integrity of compact substance on their tops is unchanged. Which diagnosis is the most likely? A. Initial parodontosis B. Atrophic gingivitis C. Initial periodontitis D. I degree periodontitis E. I degree parodontosis 4. A 26-year-old patient needs an inlay for a class V cavity of the 35 tooth. During the cavity preparation a prosthodontist has accidentally opened the pulp. What might be the most probable cause of this complication? A. Forming a at oor B. Forming divergent walls C. Forming convergent walls D. Forming a bevel E. Absence of an additional cavity 5. Parents of a 9-year-old boy complain about permanently open mouth of the child. External examination revealed elongation of the lower face part, non- closure of lips. Examination of the oral cavity revealed early mixed dentiti- on. Relationship of the rst permanent molars is neutral, vertical space is 5 mm. What is the most likely diagnosis? A. Open bite B. Distal occlusion C. Mesial occlusion D. Deep overbite E. Cross bite 6. After removal of dental deposit an 18- year-old patient underwent preventive examination. It revealed painless chalky spots in the precervical area on the vesti- bular surface of the 22 and 41 teeth. Result of enamel resistance test is 7. What morphological changes are typical for this disease? A. Subsurface enamel demineralization B. Changes in the mantle dentine C. Damage of dentinoenamel junction D. Supercial enamel demineralization E. Degeneratic changes of odontoblasts 7. A 32-year-old patient complains about gingival haemorrhages during eating and tooth brushing. It is known from the anamnesis that 2 weeks ago the patient got a cast metal crown. Objectively: the 27 tooth has a cast metal crown going beyond the gingival edge by 1-1,5 mm, touching causes haemorrhage. Percussion is slightly painful. What tactics is the most appropri- ate in this case? A. To remove the crown and fabricate a splint B. To administer gargling with Rotocan C. To open the crown through the masti- catory surface D. To give the patient time to get used to the prosthesis E. To remove the crown and administer treatment for marginal periodontitis 8. A 50-year-old patient complains about missing of the 37, 36, 45, 46, 47 teeth. The 35, 38, 44, 48 teeth are intact and stable, their crowns exhibit anatomic form, sufcient height, relative paralleli-

Upload: eneutron

Post on 14-Apr-2017

296 views

Category:

Health & Medicine


18 download

TRANSCRIPT

Page 1: Krok 2 - 2009 Question Paper (Stomatology)

Krok 2 Stomatology 2009 1

1. A 45-year-old female patientcomplains about pain caused by thermalsrtimuli and spontaneous pain in the 26tooth. A week ago this tooth was treatedfor pulpitis. Objectively: the 26 tooth isfilled, percussion is painful, thermal sti-mulus causes slowly intensifying long-lasting pain. X-ray picture shows that thepalatine canal is filled by 2/3, buccal canalscontain no filling material. What is themost probable cause of this complication?

A. Incomplete pulp extirpationB. Inadequate canal fillingC. InfectionD. Inflammation in the periodontiumE. Trauma of periodontium tissue

2. Preventive examination of a 7-year-old schoolboy revealed unremovablegrey-and-white layerings on the mucousmembrane of cheek along the line ofteeth joining. Mucous membrane is sli-ghtly hyperaemic, painless on palpation.The boy is emotionally unbalanced, bi-tes his cheeks. What is the most likely di-agnosis?

A. Mild leukoplakiaB. Chronic recurrernt aphthous stomatitisC. Chronic candidous stomatitisD. Lichen ruber planusE. Multiform exudative erythema

3. A 48-year-old patient complains aboutitching gums. Objectively: gums aredense, of normal colour; cold watercauses short-term toothache. X-ray pi-cture shows osteosclerosis of interalveolarsepta (fine granular bone texture), hei-ght of interalveolar septa and integrityof compact substance on their tops isunchanged. Which diagnosis is the mostlikely?

A. Initial parodontosisB. Atrophic gingivitisC. Initial periodontitisD. I degree periodontitisE. I degree parodontosis

4. A 26-year-old patient needs an inlay fora class V cavity of the 35 tooth. Duringthe cavity preparation a prosthodontisthas accidentally opened the pulp. Whatmight be the most probable cause of thiscomplication?

A. Forming a flat floorB. Forming divergent wallsC. Forming convergent wallsD. Forming a bevelE. Absence of an additional cavity

5. Parents of a 9-year-old boy complainabout permanently open mouth of thechild. External examination revealedelongation of the lower face part, non-closure of lips. Examination of the oralcavity revealed early mixed dentiti-on. Relationship of the first permanentmolars is neutral, vertical space is 5 mm.What is the most likely diagnosis?

A. Open biteB. Distal occlusionC. Mesial occlusionD. Deep overbiteE. Cross bite

6. After removal of dental deposit an 18-year-old patient underwent preventiveexamination. It revealed painless chalkyspots in the precervical area on the vesti-bular surface of the 22 and 41 teeth.Result of enamel resistance test is 7. Whatmorphological changes are typical for thisdisease?

A. Subsurface enamel demineralizationB. Changes in the mantle dentineC. Damage of dentinoenamel junctionD. Superficial enamel demineralizationE. Degeneratic changes of odontoblasts

7. A 32-year-old patient complains aboutgingival haemorrhages during eating andtooth brushing. It is known from theanamnesis that 2 weeks ago the patientgot a cast metal crown. Objectively: the 27tooth has a cast metal crown going beyondthe gingival edge by 1-1,5 mm, touchingcauses haemorrhage. Percussion is slightlypainful. What tactics is the most appropri-ate in this case?

A. To remove the crown and fabricate asplintB. To administer gargling with RotocanC. To open the crown through the masti-catory surfaceD. To give the patient time to get used tothe prosthesisE. To remove the crown and administertreatment for marginal periodontitis

8. A 50-year-old patient complains aboutmissing of the 37, 36, 45, 46, 47 teeth.The 35, 38, 44, 48 teeth are intact andstable, their crowns exhibit anatomicform, sufficient height, relative paralleli-

Page 2: Krok 2 - 2009 Question Paper (Stomatology)

Krok 2 Stomatology 2009 2

sm. Fulcrum line typically goes throughthe 35 and 44 abutment teeth. What fixingelement should be used in a clasp denturesupported by the 35, 44 teeth?

A. Acker claspB. Roach claspC. Jackson claspD. Telescopic fixationE. Bent wire clasp

9. A 25-year-old patient complains abouta cosmetic defect in the frontal part ofhis upper jaw on the left. Objectively: the23 tooth is pulpless. The filling restoresvestibular-approximal surface of the toothcrown by 1/3, it is stable, closely fits wi-thin cavity walls, its colour doesn’t matchthe colour of the tooth. The tooth itselfhas changed its colour, its percussion ispainless. What is the optimal prostheticconstruction for this patient?

A. Metal-ceramic crownB. Elastic crownC. Metal swaged crownD. Portion crownE. Inlay

10. A 12-year-old patient complains aboutgingival haemorrhage and tooth mobility.He has been suffering from this since theage of 4. Objectively: gums around all theteeth are hyperemic and edematic, bleedduring instrumental examination. Toothroots are exposed by 1/3 and covered withwhitish deposit. II degree tooth mobilityis present. Dentogingival pouches are 4-5mm deep. External examination revealeddryness and thickening of superficial skinlayer on the hands and feet, there are alsosome cracks. What is the most likely di-agnosis?

A. Papillon-Lefevre syndromeB. Hand-Schuller-Christian diseaseC. Generalized periodontitisD. Letterer-Siwe diseaseE. Localized periodontitis

11. A 39-year-old patient needs a claspdenture with porcelain teeth for the lowerjaw. What method should be applied whi-le substituting wax with plastic in orderto avoid mold flash that may cause biteopening?

A. Method of transfer moldingB. Direct method of compression moldingC. Inverse method of compression moldingD. Combined method of compressionmoldingE. In water under pressure

12. A 20-year-old patient complains aboutfeeling spontaneous undurable localisedpain in the 26 tooth for one day. Objecti-vely: the 26 tooth has a deep Black’s classI carious cavity. Dentine of its walls andfloor is unpigmented, softened, cariouscavity is not communicating with the toothcavity. Probing causes acute pain focusedat a single point, cold test induces long-lasting pain. What method of treatment isthe most appropriate in this case?

A. Biological methodB. Vital amputationC. Vital extirpationD. Devital extirpationE. Devital amputation

13. A 42-year-old patient complainsabout acute pain in the region of theleft temporomandibular joint (TMJ) thatirradiates to the ear; headache, generalindisposition, impossible masticationand limited mouth opening. Objecti-vely: the patient’s face is asymmetricdue to the edema in the region of theleft temporomandibular joint. The skinin this region is hyperemic. The pain ismade worse by the smallest movementsof mandible. Palpation of the joint causesacute pain. Mouth opening is limiteddown to 15-20 mm. What is the most li-kely diagnosis?

A. Acute arthritis of the left TMJB. Acute purulent parotitisC. Mandible subluxationD. Deforming arthrosis of the left TMJE. Myogenous osteoarthrosis

14. A 42-year-old patient complains aboutpain in the submandibular and sublingualregion that is getting worse during eati-ng; body temperature rise up to 37, 6oC.The patient has been suffering from thisfor 2 months. Objectively: along the ri-ght sublingual plica there is infiltration,hyperaemia, edema of soft tisues, acutepain on palpation. The duct of the rightsubmandibular salivary gland dischargesturbid saliva with pus admixtures. What isthe most likely diagnosis?

Page 3: Krok 2 - 2009 Question Paper (Stomatology)

Krok 2 Stomatology 2009 3

A. Exacerbation of urolithiasisB. Acute purulent lymphadenitis ofsubmandibular regionC. Adenophlegmon of submandibularregionD. Abscess of alveololingual grooveE. Retention cyst of sublingual salivarygland

15. A 4-year-old girl presents with bodytemperature rise, aggravation of generalcondition. The symptoms has beenobserved for 3 days. Objectively: generalcondition is grave, body temperature is38, 6oC, the girl is anxious and pale. Shepresents also with halitosis, hyperaemiaand edema of gingival mucous membranein the region of the 83, 84, 85 teeth onboth sides from the alveolar process.The mentioned teeth are mobile, theirpercussion causes acute pain; the 84 toothis filled. What is the most likely diagnosis?

A. Acute odontogenous mandibularosteomyelitis beginning from the 84 toothB. Acute sialoadenitis of submandibularsalivary glandC. Exacerbation of chronic periodontitis ofthe 84 toothD. Suppuration of the radiculodentalmandibular cyst beginning from the 84toothE. Acute odontogenous mandibular peri-ostitis beginning from the 84 tooth

16. A 38-year-old female patient came toa hospital and complained about a woundon her left cheek. She got this trauma 16-18 hours ago, didn’t loose consciousness.Objectively: a 3 cm long injury of skin,subcutaneous fat and muscle. The woundis bleeding. What kind of initial surgicald-bridement should be performed?

A. Early surgical d-bridementB. Delayed surgical d-bridementC. Secondary surgical d-bridementD. Initial surgical d-bridement along withplastyE. -

17. A 47-year-old patient consulted adentist about dental cervix exposure onboth jaws. Objectively: the dentitions areintact, the dental cervixes are exposed.Untimely teeth contacts are present. Itis planned to perform selective grindi-ng. What controlling method should bechosen?

A. OcclusiographyB. MasticatiographyC. Chewing testD. RoentgenographyE. Gnathodynamometry

18. During extraction of the 47 tooth itsdistal root was broken halfway along itslength. What tool should be chosen forextraction of the residual root fragments?

A. Left angled elevatorB. Broad-beaked forcepsC. Close-beaked forcepsD. Right angled elevatorE. Straight elevator

19. A 31-year-old patient consulted adentist about teeth sanitation. Objecti-vely: in the precervical region of the 13and 23 teeth there are irregularly-shapeddefects of hard tissues within deep layersof enamel. The floor of the defects isrough, their edges are chalky. What tacticsshould be chosen as to these defects?

A. Preparation and fillingB. Filling with composite material withoutpreparationC. Remineralizing therapyD. Grinding of the defectsE. Silvering of the defects

20. A 51-year-old female patient complai-ns about food sticking in a right inferi-or tooth. Objectively: distal masticatorysurface of the 45 tooth has a deep cari-ous cavity filled with dense pigmenteddentin that doesn’t communicate with thetooth cavity. The patient was diagnosedwith chronic deep caries. What method ofexamination allowed the dentist to elimi-nate chronic periodontitis?

A. Electro-odontometryB. ProbingC. Palpation of projection of root apexD. PercussionE. Cold test

21. An 18-year-old patient complainsabout body temperature rise, weakness,pain induced by eating and deglutition.Objectively: mucous membrane of theoral cavity is erythematic with multi-ple petechia. Pharynx is hyperaemic.Regional lymph nodes are enlarged,mobile, painless. In blood: leukocytosis,monocytosis, atypic mononuclears, ESRis 30 mm/h. What is the leading factor ofdisease development?

Page 4: Krok 2 - 2009 Question Paper (Stomatology)

Krok 2 Stomatology 2009 4

A. Viral infectionB. Bacterial infectionC. Autoimmune disordersD. Immediate allergyE. Delayed allergy

22. A female patient consulted astomatologist about inability to close hermouth, speech difficulty. Objectively: theoral cavity is half-open, there is drooling,central line deviates to the right. Thereis a depression in front of the tragus,and above the malar arch the mandibularcondyle bulges inward the infratemporalfossa. What is the most likely diagnosis?

A. Unilateral anterior dislocation ofmandible on the leftB. Unilateral anterior dislocation ofmandible on the rightC. Unilateral posterior dislocation ofmandible on the leftD. Unilateral posterior dislocation ofmandible on the rightE. Bilateral anterior dislocation

23. Analysis of a 10-year-old boy’s jawmodels revealed that occlusal plane ofthe frontal maxillary teeth was of concaveform, its lateral parts were convex. Formof the alveolar process also representsdeformation of dental arches. The upperjaw is of saddle-like form with abruptnarrowing in the region of premolar teeth.What type of bite is it?

A. OpenB. DistalC. DeepD. MesialE. Cross

24. A 50-year-old patient complains aboutincreased sensibility of the exposed toothcervixes, displacement of teeth, gum itch,pain in the region of the 43, 42, 41,31, 32, 33 teeth caused by chemical andthermal stimuli. Objectively: the gums aredense and anemic. X-ray picture showsatrophy of the alveolar bone reaching 2/3of interalveolar septa height. What is themost likely diagnosis?

A. III degree parodontosisB. II degree parodontosisC. Chronic generalized III degreeparodontosisD. Chronic generalized II degreeparodontosisE. Atrophic gingivitis

25. A 32-year-old patient complains ofa fistula in the submandibular region.

Objectively: cheek is swollen in the regi-on of mandible on the left, palpationrevealed induration of soft tissues. Inprojection of the 35, 36 teeth there is afistula containing pus and granulations.X-ray picture shows destruction of bone,sequestrum. What is the most likely di-agnosis?

A. Chronic osteomyelitisB. ActinomycosisC. SyphilisD. SarcomaE. Osteoma

26. A 23-year-old patient complains abouta small ulcer on the red border of herlower lip that has been irresponsive toself-treatment for two weeks. Objectively:unchanged red border of lower lip has acircular ulcer of 2 mm in diameter withraised regular edges, its floor is of meat-like colour, dense, shiny, with "stearic fi-lm", with cartilaginoid infiltration, pai-nless on palpation. Regional lymph nodeis enlarged, of tight elastic consistency,painless, mobile. What is the most likelydiagnosis?

A. Primary syphilisB. Decubital ulcerC. Cancerous ulcerD. Lupus erythematosusE. Tuberculous ulcer

27. X-ray picture depicts a circular well-defined area of bone tissue destruction0,7х0,7 cm large in the projection of rootapex. What is the most likely diagnosis?

A. CystogranulomaB. CystC. GranulomaD. OdontomaE. Osteoma

28. A 29-year-old patient complains aboutacute attack-like pain in the region ofhis upper jaw on the left, as well as inthe region of his left maxillary sinus, eyeand temple. The pain is long-lasting (2-3hours), it is getting worse at night. Thepatient has a history of recent acute respi-ratory disease. Objectively: the 26 toothhas a carious cavity, floor probing is pai-nful, thermal stimuli cause long-lastingpain, percussion causes slight pain. Whatis the most likely diagnosis?

Page 5: Krok 2 - 2009 Question Paper (Stomatology)

Krok 2 Stomatology 2009 5

A. Acute diffuse pulpitisB. Acute focal pulpitisC. Acute apical periodontitisD. Inflammation of maxillary sinusE. Exacerbation of chronic periodontitis

29. A 45-year-old patient complains aboutpain in his mandible that arose afterextraction of the 36 tooth. Objectively:alveolar socket is covered with bloodyclot. X-ray picture shows unextracted rootof the 36 tooth. What tools are necessaryfor extraction of this root?

A. Angled elevator curved rightB. Angled elevator curved leftC. Straight elevatorD. S-shaped forcepsE. Bayonet-shaped forceps

30. A patient complains of pain and swelli-ng in the right submandibular area. Shehas been treating the 45 tooth for a week.Objectively: body temperature is 38oC.There is a painful tense infiltration inthe right submandibular region. The skindoesn’t make a fold, its hyperemic andglossy. The mouth can be opened by 3 cm.Deglutition is painless. These clinical fi-ndings correspond with the following di-sease:

A. Odontogenous phlegmon of the rightsubmandibular regionB. Abscess of the right alveololingualgrooveC. Adenophlegmon of the right submandi-bular regionD. Acute odontogenous sialoadenitisE. Phlegmon of pterygomandibular space

31. A 5-year-old boy presents with bodytemperature rise up to 39, 2oC, sorethroat, headache, nausea. Objectively:mucous membrane of soft palate andpalatine arches is evidently hyperemic,there is distinct border between it andsurrounding tissues. The child’s tongue isdry, edematic, its lateral surfaces are redand free of fur, fungiform papillae are evi-dently enlarged. Face skin is covered wi-th spotty rash, hyperemic except for palenasolabial triangle. Submandibular lymphnodes are painful on palpation. What isthe causative agent of this disease?

A. Haemolytic streptococcusB. Coxsackie virusC. Herpes virusD. Bordet-Gengou bacillusE. Loffler’s bacillus

32. Preventive examination of a 4,5-year-

old child revealed untimely missing ofall the upper molars. The lower incisorscontact with mucous membrane of palate.What is the tactics of choice?

A. Fabrication of a removable lamellarprosthesisB. Fabrication of an orthodontic appliancefor deep overbite correctionC. Annual examination till cutting of thepermanent teethD. Half-yearly examination till cutting ofthe permanent teethE. Medical intervention is not required

33. A 20-year-old patient complains abouta carious cavity in an upper right tooth.Objectively: the 16 tooth has a deep cari-ous cavity communicating with the toothcavity, probing at the opening point is pai-nless, percussion of the 16 causes mildpain. There is a fistula on the gingiva inthe region of root apex projection of the16 tooth. What is the most probable di-agnosis?

A. Chronic granulating periodontitisB. Chronic fibrous periodontitisC. Chronic granulomatous periodontitisD. Chronic hypertrophic pulpitisE. Chronic gangrenous pulpitis

34. A 16-year-old patient complains abouta cosmetic defect in form of white spots inthe region of the upper frontal teeth. Thedefect was revealed long ago and doesn’tchange with time. Objectively: white spotson the vestibular surfaces of the 11, 12, 21,22 teeth close to the cutting edge and onthe vestibular surfaces of the 16, 26, 36, 46teeth close to the masticatory surface. Onprobing the spot surface was smooth, pai-nless; cold stimulus produced no pain. Thespots couldn’t be stained with 2% solutionof methylene blue. What is the most likelydiagnosis?

A. Systemic enamel hypoplasiaB. Local enamel hypoplasiaC. Acute initial cariesD. Fluorosis in form of spotsE. Erosion of dental solid tissues

35. A 35-year-old patient complains abouta cavity in a lower jaw tooth on the left,and the pain caused by sweet, sour andsolid food. Examination of the 36 toothrevealed a deep carious cavity filled withlight softened dentine. Probing of the cari-ous cavity floor is painful, reaction to thecold stimulus is painful, undurable. Whatis the most likely diagnosis?

Page 6: Krok 2 - 2009 Question Paper (Stomatology)

Krok 2 Stomatology 2009 6

A. Acute deep cariesB. Acute focal pulpitisC. Chronic fibrous pulpitisD. Acute median cariesE. Chronic deep caries

36. A 31-year-old male patient complai-ns of dryness and burning of tongueback that appeared for about a weekago and get worse when he eats irritati-ng food. The patient has a history ofrecent pneumonia. He had been treatedin the in-patient hospital for 2 weeks, thetreatment program included antibiotics.Now he doesn’t take any drugs. Objecti-vely: mucous membrane of the oral cavityis hyperemic, dry, glossy. Tongue back andpalate have greyish-white plicae that canbe easily removed. Threads of saliva trailbehind the spatula. What is the most likelydiagnosis?

A. Acute pseudomembranous candidiasisB. Chronic hyperplastic candidiasisC. Acute atrophic candidiasisD. Medicamental stomatitisE. Chronic atrophic candidiasis

37. A 53-year-old patient consulted aprosthodontist about lateral teeth mobili-ty, frequent falling out of fillings. Objecti-vely: all the molars and premolars ofthe lower jaw exhibit I degree mobility.Approximal masticatory surfaces have fi-llings. What splint construction should beapplied in this case?

A. Inlay splintB. Fixed crown splintC. Crown cap splintD. Equator crown splintE. Intradental splint

38. A 28-year-old patient consulted aprosthodontist about dental prostheti-cs. Objectively: crown of the 13 tooth isdecayed by 2/3 of its height, the stump sti-cks out above the gum line by 2 mm. X-raypicture shows that the root canal is filledup to the top. What construction shouldbe fabricated for the patient?

A. Pivot toothB. CrownC. InlayD. Equator crownE. The crown should be restored withphotopolymer

39. A 65-year-old patient complains aboutpartially missing teeth on his upper jaw,difficult mastication, rhinolalia. Objecti-vely: the 18, 16, 15, 11, 23, 28, 35, 38, 48,

47 teeth are missing; there is postoperati-ve midline defect of hard palate. It wasdecided to make a clasp dental prosthesiswith obturating part. The obturating partshould be placed on the following elementof the clasp dental prosthesis:

A. On the archB. On the saddleC. On the artificial teethD. On the baseE. On the clasps

40. A 10-year-old boy consulted a denti-st about pain in the palate during eating.Objectively: the lower third of his face isshortened, mouth opening is not limited.By joining the teeth the cutting edge ofinferior incisors contacts with the mucousmembrane of palate. Mucous membranein the contact point is hyperemic, slightlyedematic. Lateral teeth exhibit Angle’sclass I malocclusion. What is the mostappropriate plan of treatment of the lowerjaw?

A. To impact frontal partB. To impact lateral partsC. To widen the lower jawD. To protract frontal partE. To protract lateral parts

41. A patient consulted an oral surgeonabout pain in the region of his left upperjaw that appeared 3 days ago. Afterexamination the patient was diagnosedwith exacerbation of chronic periodontitisof the 17 tooth. It is indicated to extractthe 17 tooth. What nerves should beblocked for painless extraction of the 17tooth?

A. Posterior alveolar nerves and greaterpalatine nerveB. Greater palatine nerveC. Anterior alveolar nerves and incisornerveD. Median alveolar nerves and greaterpalatine nerveE. Nasopalatine nerve

42. A 40-year-old patient complains aboutfrequent falling out of a filling. Objecti-vely: the 46 tooth has a carious (Black’sclass II) cavity. It is planned to restore theanatomic form of the tooth by means ofa metal inlay. What is the peculiarity ofthe cavity preparation for the inlay in thiscase?

Page 7: Krok 2 - 2009 Question Paper (Stomatology)

Krok 2 Stomatology 2009 7

A. Forming a bevelB. Forming an additional shoulderC. Cavity floor wideningD. Preparation for parapulpar postsE. Cavity deepening

43. A 10-year-old child complains aboutacute spontaneous spasmodic pain in anupper jaw tooth on the left. Objectively:distal contact surface of the 26 toothexhibits a carious cavity filled with li-ght softened dentine and localized withinparapulpar dentine. Probing of the cavityfloor causes acute pain, percussion is pai-nless. Cold stimuli cause a long-standingpain attack. The child has a history of li-docaine allergy. Choose an optimal pasteto be used during the first visit:

A. ParaformaldehydeB. IodoformC. ThymolD. FormocresolE. Zinc oxide eugenol

44. A 35-year-old patient complains aboutitch, burning and edema of lips. He hasbeen suffering from this for a week.Objectively: reddening of red border andskin, especially in the region of mouthcorners, there are also vesicles, crusts,small cracks along with erythematousaffection of red border. What is the mostlikely diagnosis?

A. Acute eczematous cheilitisB. Multiform exudative erythemaC. Acute herpetic cheilitisD. Allergic contact cheilitisE. Exudative form of exfoliative cheilitis

45. Preventive examination of an 8-year-old boy revealed some lusterless chalk-like spots on the vestibular surface of the11 and 21 teeth, which are localised in theprecervical region. Subjective complaintsare absent. What is the most likely di-agnosis?

A. Acute initial cariesB. White-spotted fluorosisC. Local enamel hypoplasiaD. Acute superficial cariesE. Chronic initial caries

46. A 27- year-old patient complainsabout acute pain in the region of the34 tooth that is getting worse when bi-ting down on food. Roentgenographicalsurvey revealed an ill-defined zone ofbone tissue destruction in the periapicalregion of root of the 34 tooth. What is themost likely diagnosis?

A. Exacerbation of chronic granulatingperiodontitisB. Acute purulent pulpitis complicated byperiodontitisC. Exacarbation of chronic pulpitisD. Exacerbation of chronic granulomatousperiodontitisE. Acute serous periodontitis

47. A patient undergoes orthopaedictreatment of bounded edentulous spaceson the upper jaw. He needs fixed full-cast dentures. During his second visit itis required to check whether the internalsurface of the metal framework of thefuture metal-ceramic denture matches thesurfaces of the prepared teeth. In whatway could this be done?

A. In the oral cavity by means of siliconematerialsB. Visually by means of models in thearticulatorC. In the oral cavity by means of tracingpaperD. In the oral cavity by means of a waxplateE. In the oral cavity by means ofstomatoscopic method

48. An 18-year-old female patientconsulted a dentist about dental prostheti-cs. Objectively: the 21 tooth is pulpless,of dark-grey colour. The patient hasorthognathic occlusion. It is planned tocrown the tooth with a plastic crown.Choose the plastic mass for its fabrication:

A. SinmaB. EthacrylC. AcrelD. FtoraxE. Bakril

49. A 47-year-old patient complains aboutpermanent pain in the 27 tooth thatis getting worse when biting down onfood. Objectively: the patient’s face issymmetric, skin is of normal colouring,the mouth can be fully opened, mucousmembrane of the alveolar process isedematic and hyperemic at a level of the27 tooth. The 27 tooth has a deep cariouscavity interconnecting with pulp chamber.Percussion of the 27 tooth causes acutepain. What is the most likely diagnosis?

Page 8: Krok 2 - 2009 Question Paper (Stomatology)

Krok 2 Stomatology 2009 8

A. Exacerbated chronic periodontitis ofthe 27 toothB. Chronic periodontitis of the 27 toothC. Acute generalized purulent pulpitis ofthe 27 toothD. Acute purulent periostitis of the upperjaw beginning from the 27 toothE. Chronic left-sided odontogenous hi-ghmoritis

50. A 64-year-old female patient was wai-ting for her turn at the dentist’s. Suddenlyshe fell down, her respiration becamehoarse, she got convulsive twitching inher upper and lower limbs, face and neckturned cyanotic, eye pupils became mydri-atic, reaction of eye to light was absent.Arterial pressure and pulse couldn’t bemeasured. Heart sounds couldn’t beauscultated. Involuntary urination wasnoted. What condition is characterized bysuch symptoms?

A. Clinical deathB. EpilepsyC. ShockD. ComaE. Collapse

51. A 3-year-old child complains of a cavi-ty in a lower jaw tooth. Objectively: masti-catory surface of the 84 tooth exhibitsa carious cavity within mantle dentine.Dentine is softened, probing of the cavi-ty walls is painful, percussion is painless.What is the most likely diagnosis?

A. Acute median cariesB. Chronic superficial cariesC. Chronic fibrous pulpitisD. Acute deep cariesE. Chronic deep caries

52. A 43-year-old patient complainsabout a neoplasm in the right submandi-bular region that appeared a month agoafter angina. Body temperature is up to37, 0 − 37, 2oC. The patient underwentanti-inflammatory therapy but it led to noreduction of the neoplasm. Objectively: inthe right submandibular region palpati-on reveals a slightly painful, spherical,freely movable, well-defined neoplasm ofdense and elastic consistency. The duct ofsubmandibular salivary gland dischargestransparent saliva. The sublingual plica isunchanged. What is the most likely di-agnosis?

A. Chronic lymphadenitisB. Chronic sialoadenitisC. SialolithiasisD. Adenoma of salivary glandE. Atheroma

53. A 38-year-old patient ordered aclasp upper jaw prosthesis. Its fabricati-on involves forming of a fireproof model.What material should be applied forduplication?

A. GelinB. StomatoplastC. StomalginD. OrthocorE. Dentafol

54. A 35-year-old patient needs a metalinlay for the 37th tooth. Objectively:masticatory surface of the 37 tooth hasa carious cavity. What is the peculiarity oftooth preparation?

A. Forming a bevelB. Forming an additional shoulderC. Widening of cavity floorD. Forming flat floorE. Forming additional cavity

55. A 64-year-old patient complains abouta slightly painful ulcer on the lateralsurface of his tongue that appeared over 2months ago. Objectively: lateral surface oftongue has an ulcer with raised edges, ti-ssues around it are infiltrated. Submandi-bular lymph nodes are enlarged, adheri-ng to each other and surrounding tissues.What is your provisional diagnosis?

A. Lingual cancerB. Syphilis (hard chancre)C. Tuberculous ulcerD. Decubital ulcerE. Ulcero-necrotic lesion associated withblood disorders

56. A 56-year-old patient complains aboutmissing lateral teeth on both sides ofmandible. Objectively: the 48, 47, 46, 45,35, 36, 37, 38 teeth are missing. Make a di-agnosis:

A. Kennedy I denttion defectB. Kennedy IV denttion defectC. Kennedy II denttion defectD. Kennedy III denttion defectE. Betel’man II denttion defect

57. A 4-year-old child got a face trauma 2hours ago. A stomatologist on duty madea diagnosis: intrusive luxation of the 61tooth. What is the tactics of choice?

Page 9: Krok 2 - 2009 Question Paper (Stomatology)

Krok 2 Stomatology 2009 9

A. Extraction of the 61 toothB. ObservationC. Reposition of the 61 toothD. Splinting of the 61 toothE. Removal of pulp of the 61 tooth

58. An 18-year-old student complainsof roughness and dryness of mucousmembrane of the oral cavity. He always bi-tes uncontrolledly off mucous membranealong the line of teeth joining. The pati-ent suffers from chronic cholecystitis, heis smoker. Hygiene index is 2,3. Objecti-vely: mucous membrane along the line ofteeth joining is quaggy, edematic, whitish.What is the leading factor of this diseasedevelopment?

A. Habitual biting of mucous membraneof the oral cavityB. Unsatisfactory level of oral cavityhygieneC. Somatic pathologyD. XerostomiaE. Smoking

59. A 57-year-old patient complains aboutdryness and burning of the lateral surfaceof her tongue. These sensations disappearduring eating. She noted such sensationsthree months ago. She has a history ofgastritis with reduced secretory function.Objectively: mucous membrane of tongueand oral cavity has no peculiarities. Theback of tongue has thin white coating.Regional lymph nodes are unpalpable.Oral cavity is sanitized. What is the mostlikely diagnosis?

A. GlossodyniaB. Lingual nerve neuritisC. CandidiasisD. Desquamative glossitisE. Hunter-Moeller glossitis

60. A 23-year-old patient complainsabout a carious cavity in the 15 tooththat he noted a year ago. Examinati-on revealed that approximal surface hada deep carious cavity not communi-cating with the tooth cavity and fi-lled with dense pigmented dentine.Probing, cold test, percussion cause nopain. Electroodontodiagnosis is 10 mi-croampere. What is the most likely di-agnosis?

A. Chronic deep cariesB. Chronic fibrous periodontitisC. Chronic fibrous pulpitisD. Chronic median cariesE. Chronic granulomatous periodontitis

61. A 7-year-old child complains aboutspontaneous pain in the upper rightmolars. Both medial contact and masti-catory surfaces of the 55 tooth have acarious cavitiy filled with clear softeneddentin and localized within parapulpardentin. Floor probing causes acute pain,tooth percussion is slightly painful.Mucous membrane of the alveolar processin the root projection of the 55 is intact,examination of this region revealed noroentgenological changes. What is themost likely diagnosis?

A. Acute diffuse pulpitisB. Chronic fibrous pulpitisC. Chronic gangrenous pulpitisD. Exacerbation of chronic periodontitisE. Deep acute caries

62. A 55-year-old patient has a repeatedappointment with a dentist. He hasordered complete removable lamellardentures for both upper and lower jaw.During his previous visit the dentisttook complete anatomic impressions ofboth jaws and sent them to the dentalmechanic. What manipulations should thedentist perform at the next clinical stage?

A. Adjust individual traysB. Locate and fix central occlusionC. Check the prosthesis constructionwithin the oral cavityD. Correct the prosthesis and give thepatient necessary instructionsE. Fabricate occlusal rims

63. A 33-year-old patient complai-ns about pain, gingival haemorrhage,halitosis, body temperature rise upto 37, 8oC. Objectively: gums areapparently hyperaemic, edematic, bleedeasily, parodontal pouches are 3-4 mm deep, and contain purulentexudate. Orthopantomogram shows di-ffuse osteoporosis of alveolar process,resorption of interdental septa down to1/3 of their height. What is the most likelydiagnosis?

Page 10: Krok 2 - 2009 Question Paper (Stomatology)

Krok 2 Stomatology 2009 10

A. Exacerbation of chronic generalized Idegree periodontitisB. Chronic generalized I degree peri-odontitisC. Exacerbation of chronic generalized IIdegree periodontitisD. Chronic generalized II degree peri-odontitisE. Exacerbation of chronic catarrhal gingi-vitis

64. An 8-year-old boy complains aboutacute short-term pain in the 11 toothcaused by cold and sweet stimuli. Objecti-vely: medial contact surface of the 11tooth has a carious cavity within denti-noenamel junction filled with softenedmoist dentine that can be easily removed.Enamel edges are white, undermined,friable. Superficial probing is painless,thermal test is positive, percussion isnegative. What is the filling material ofchoice?

A. Glass ionomer cementB. Silicophosphate cementC. Composite materialD. Silver amalgamE. Silicate cement

65. A 40-year-old patient complains aboutbody temperature rise up to 38oC, andof a roundish infiltration on his upper lip.Objectively: there is a roundish infiltrationon the upper lip on the left, the skin aboveit is wine red, the infiltration adheres tothe surrounding tissues and has a seapusnecroticus in the middle. The upper lipis hyperaemic and edematic. What is themost likely diagnosis?

A. Upper lip furuncleB. Acute periostitis of the upper lipC. Retention cystD. Acute lymphadenitisE. Upper lip carbuncle

66. A female patient complains about painin the 11 tooth caused by sour and sweetfood. Objectively: enamel changes in formof chalky appearance, a defect with lightbottom within dentinoenamel junction onthe vestibular surface in the precervicalarea of the 11 tooth. Probing was painless,percussion and cold stimulus caused nopain. What is the most likely diagnosis?

A. Acute superficial cariesB. Acute initial cariesC. Acute median cariesD. Enamel hypoplasiaE. Fluorosis

67. A 69-year-old patient needs extractionof the 12, 11 teeth. He is diagnosed withgeneralized periodontitis, the 12 and 11teeth exhibit II degree mobility. Choose aproper instrument for extraction:

A. Straight forcepsB. S-shaped forcepsC. S-shaped forceps curved rightD. Root bayonet-shaped forcepsE. Crown bayonet-shaped forceps

68. An 8-year-old child presents with anedema of the submandibular region, themouth can be opened by 1,5 cm, furtheropening is difficult, body temperature is37, 6oC, mucogingival fold is vestibularlyflattened, hyperaemic and swollen. The 84and 85 teeth have fillings, their percussi-on is painless. The 84 tooth exhibits Idegree mobility. What is the most likelydiagnosis?

A. Acute odontogenic periostitisB. Acute odontogenic osteomyelitisC. Chronic odontogenic periostitisD. Chronic odontogenic osteomyelitisE. Exacerbation of chronic periodontitis

69. A 25-year-old patient has a medianfracture of mandible without evident di-splacement of fragments. All the teeth areintact. What kind of wire splint is the mostappropriate in this case?

A. Smooth splintB. Smooth splint with a spreading curveC. Splint with hooksD. Splint with guide planeE. Pomerantseva-Urbanskaya’s splint

70. A 19-year-old patient complains aboutpain in the submental part of mandible.The day before he got a trauma. Objecti-vely: there is a slight swelling of tissuesin the mental area. The mouth can beopened widely enough. All the teeth areintact. Mucous membrane is edematic inthe region of central incisors, it bleeds sli-ghtly. In this region mobility of mandiblefragments is present. Occlusion is undi-sturbed. What splint should be chosen bythe dentist?

A. Smooth braceB. Splint with spreading curveC. Splint with guide planeD. Anchor splint with intermandibularfixationE. Ivy loops for 31 32, 42 41

71. A 25-year-old man complains of itchi-ng and reddening of skin in the buccal

Page 11: Krok 2 - 2009 Question Paper (Stomatology)

Krok 2 Stomatology 2009 11

area, general weakness, inertness. Heassociates origin of the disease with askin injury he got during shaving. Objecti-vely: body temperature is 39, 0oC. A well-defined section of skin in the buccal areais hyperemic, it slightly protrudes abovethe surface. Hyperemic surface has somevesicles containing serous fluid. What isthe most likely diagnosis?

A. Erysipelatous inflammation of skinB. Phlegmon of buccal areaC. FuruncleD. AnthraxE. Streptococcal impetigo

72. A 30-year-old saxophonist complai-ns about wearing of the occlusal surfaceof his frontal teeth along with separati-on between the upper and lower teeth(apertognathia). Height of the lower thirdof the patient’s face remains unchanged.What method should be applied for elimi-nation of this deformity?

A. ProteticB. Increase of the interalveolar heightC. Shortening of teethD. OrthodonticE. Instrumental-and-surgical

73. A 32-year-old patient complains aboutmouth soreness, body temperature riseup to 38, 5oC, indisposition. Such condi-tion has been occurring periodically forseveral years after the patient had had acold. Objectively: lips are covered withhaemorrhagic crusts, hyperaemic mucousmembrane of lips and cheeks has erosionscovered with fibrinous films. Hypersali-vation is present. What is the most likelydiagnosis?

A. Multiform exudative erythemaB. Pemphigus vulgarisC. Herpes recidivicusD. Herpetiform Duhring’s dermatitisE. Stevens-Johnson syndrome

74. Examination of a patient revealed aroundish neoplasm in the submental area.Skin above it is unchanged, makes a foldeasily. Puncture sample contains strow-yellow liquid with cholesterol admixture.What is the most likely diagnosis?

A. Inborn median cystB. LipomaC. LymphadenitisD. Retention cyst of sublingual salivaryglandE. Dermoid cyst

75. A 13-year-old girl complains aboutfrequent falling out of a filling in the 21tooth. It is known from the anamnesis that2 years ago she underwent treatment onaccount of a dental trauma. Objectively:a transverse defect of 1/3 of the crown inthe 21 tooth. Percussion is painless. Toothcolour is unchanged. X-ray picture showsthat root canal is filled by 1 mm from theapex, filling material closely fits the wallsof root canal. What stomatological tacticsshould be chosen?

A. The defect should be restored withphotopolymerB. The root canal should be refilledC. The root apex should be resectedD. The tooth should be extractedE. The tooth should be crowned with anartificial crown

76. A 35-year-old patient was diagnosedwith chronic median caries of the 36 tooth.There is a Black’s class II cavity affectingmasticatory surface. What material shouldbe chosen for the tooth filling?

A. Light-cure microhybride compositeB. Glass ionomer cementC. Silicophosphate cementD. Light-cure fluid compositeE. Light-cure microfilled composite

77. A 79-year-old female patientconsulted a prosthodontist about denturereplacement. The patient has a historyof a stroke. Objectively: acute irregularatrophy of the alveolar processes of bothjaws is present; mucous membrane ofthe oral cavity is dry and nonmobile.The previous dentures cannot be fixed.What is the most appropriate prostheticconstruction?

A. Dentures with elastic liningB. Dentures with extended bordersC. Dentures with metal basesD. Dentures with shortened bordersE. Implant-supported dentures

78. A 48-year-old teacher complainedabout considerable mobility of the 42, 41,31, 32 teeth. Objectively: cervixes of the42, 41, 31, 32 are exposed by 2/3, there ispathological III degree mobility. It is indi-cated to extract the 42, 41, 31, 32 teeth.What type of denture should be used forthe direct prosthetics?

Page 12: Krok 2 - 2009 Question Paper (Stomatology)

Krok 2 Stomatology 2009 12

A. Removable partial lamellar dentureB. Clasp dentureC. AdhesiveD. Metal-ceramic bridge-like dentureE. Swaged-and-soldered bridge-likedenture

79. Examination of a 13-year-old patientallowed to make a final diagnosis: vesti-bular position of the 13 and 23 teeth withthe total space deficit, narrowing of maxi-llary dental arch, torsion of the 12 and 22tooth. To eliminate this pathology it wassuggested to widen the dental arch andto extract some teeth. What teeth haveorthodontic indication for their extracti-on?

A. First premolarsB. CaninesC. Second incisorsD. Second premolarsE. First molars

80. Parents of a 2-year-old girl complainabout fistulas with purulent discharge inthe region of the upper frontal teeth.Objectively: crowns of the 51, 52, 61, 62teeth are significantly decayed, probingof root canal orifices is slightly painful, itcauses significant haemorrhage. Percussi-on is painless. Mucous membrane of thealveolar process is pastose and cyantotic,there are cicatrices and fistulas in this regi-on. What is the most likely diagnosis?

A. Exacerbation of chronic granulatingperiodontitisB. Exacerbation of chronic granulomatousperiodontitisC. Exacerbation of chronic fibrous peri-odontitisD. Chronic granulomatous periodontitisE. Chronic granulating periodontitis

81. A month after cementing the metal-ceramic crown on the 23 tooth a patientcomplained of its decementing. Exami-nation revealed that the tooth stump wasof sufficient height, stump walls wereconvergent to the vertical tooth axis at anangle of approximately 30 degrees. Whatangle of convergence had to be formed?

A. Up to 8 degreesB. 12-15 degreesC. 15-18 degreesD. 22-25 degreesE. 10-12 degrees

82. During lateral movements of mandi-ble the frontal teeth are displaced si-dewards. Deviation of incisor point from

the central position is measured by anangle of 100− 110o. What is characterizedby such a value of incisor point deviation?

A. Lateral incisor pathB. Lateral joint pathC. Sagittal incisor pathD. Sagittal joint pathE. Bennett’s angle

83. Parents of a 1,5-year-old chi-ld complain about the child’s tongueenlargement and ingestion disorder. Thechild has been suffering from this since bi-rthday. Objectively: general condition hasno peculiarities. The tongue is enlarged(macroglossia). Its mucous membraneexhibits granular vesicular outgrowths.The tongue is dense, painless on palpati-on. What is the most likely diagnosis?

A. Lymphangioma of tongueB. Hemangioma of tongueC. Fibroma of tongueD. Cyst of tongueE. Cancer of tongue

84. A 52-year-old patient complains ofsignificant tooth wear on both jaws.Objectively: in the lateral parts of mandi-ble teeth are worn down to the gums,in the frontal parts teeth are worn by1/3. Treatment includes two stages. Whatprosthetic construction should be appli-ed for bite disjoining and myotatic reflexchange?

A. Periodontal splint for the lateral partsB. Plastic splint for the whole dentitionC. Plastic splint for the frontal partD. Crown splint for the lateral partsE. Elastic plastic splint

85. A girl is 18 months old. Vestibularsurfaces of the 52, 51, 61, 62 teeth have wi-de carious cavities within enamel. Probingis slightly painful, percussion of the 52, 51,61, 62 is painless. What is the treatment ofchoice?

A. Silver impregnationB. Filling with amalgamC. Remineralizing therapyD. Fluorine laquer coatingE. Filling with phosphate cement

86. A 53-year-old patient consulted aprosthodontist about dental prosthetics.Objectively: the 13 and 24 teeth remainas well as all the teeth on the lower jaw.The lower third of the patient’s face isshortened, crowns of the 13, 24 teethare worn off by 2/3. What constructional

Page 13: Krok 2 - 2009 Question Paper (Stomatology)

Krok 2 Stomatology 2009 13

element will provide optimal fixation ofthe partial removable prosthesis on theupper jaw?

A. Telescopic crownsB. Compound claspsC. Retaining claspsD. BarsE. Dentoalveolar clasps

87. Parents of a 6-year-old child applied toa pedodontist for preventive examinati-on of their child. The oral cavity is sani-tized. According to the parents, the childhas recently cut the 36 and the 46 tooth.What method of caries prevention shouldbe applied within 1,5-2 years after cuttingof the mentioned teeth?

A. Fissure hermetizationB. Fissure silveringC. Coating the teeth with fluorine lacquerFtorlakD. Remodentum solution applicationsE. Gargling with sodium fluoride

88. Examination of a 6-year-old girlrevealed a deep carious cavity in the 85tooth. Percussion and probing are pai-nless. After removal of the softened denti-ne communication with the tooth cavityshowed up. Deep probing is painless. X-ray picture of the 85 tooth shows the focusof destruction of bone tissue in the regionof bifurcation; cortical plate of the 35 hasno pathological changes. It is most expedi-ent to use the following material for theroot filling:

A. Zinc oxide eugenol cementB. Resorcin-formalin pasteC. Glass ionomer cementD. Phosphate cementE. Calcium-containing paste

89. A 48-year-old patient got a pustuleon his chin that quickly developed into adense and acutely painful infiltration 3x5cm large. The skin above it is of blue-redcolour. In the centre one can see threezones of necrosis around the hair follicles.Lymph nodes of chin are enlarged andpainful. What is the most likely diagnosis?

A. Chin carbuncleB. Erysipelatous inflammation of chinC. Dermal actinomycosis of chinD. Suppurated atheromaE. Chin furuncle

90. A patient applied to the oral surgerydepartment and complained about painand edema in the right infraorbital regi-

on and right zygomatic region, skinnumbness in the area of the right half ofhis upper lip; nasal haemorrhage. Thesesymptoms turned up after a trauma. Whatdisease should be suspected?

A. Fracture of zygomatic boneB. Le Fort I maxillary fractureC. Le Fort II maxillary fractureD. Le Fort III maxillary fractureE. Fracture of nose bones

91. A 56-year-old patient has an oval,smooth, bright-red erosion on the redborder of her lower lip. Erosion is coveredwith haemorrhagic crusts that can behardly removed. Crust removal inducesslight haemorrhage. Light traumatizationof crust-free surface of erosion induces nohaemorrhage. Specify the type of lowerlip precancer:

A. Abrasive precancerous Manganotti’scheilitisB. Verrucous precancer of red borderC. Localized precancerous hyperkeratosisof red borderD. Bowen’s diseaseE. Erythroplasia

92. A 12-year-old child complains aboutbleeding from the tooth socket duringeating and tooth brushing. The toothhas hurt him before. Objectively: the 36tooth has a deep cavity communicatingwith the tooth cavity and filled with redexcrescences. Probing causes pain and sli-ght haemorrhage; percussion is painless,thermal stimuli cause mild pain. What isyour provisional diagnosis?

A. Chronic hypertrophic pulpitisB. Chronic granulating pulpitisC. Chronic papillitisD. Gingival polypE. Chronic simple pulpitis

93. A 23-year-old patient complainsabout gingival haemorrhage during toothbrushing and eating solid food. Objecti-vely: gingiva of the frontal part of mandi-ble is hyperaemic, edematic, it bleeds onpalpation. Mucous membrane of the oralcavity as well as gingiva in other regionspresent no changes. The patient has deepoverbite. Teeth are stable except for the41 and 31 (I degree of mobility). X-raypicture shows resorption of interalveolarsepta by 1/3 of root length in the region ofthe 42, 41, 32, 31 teeth. What is the mostlikely diagnosis?

Page 14: Krok 2 - 2009 Question Paper (Stomatology)

Krok 2 Stomatology 2009 14

A. Localised periodontitisB. Initial generalized periodontitisC. Generalized I degree periodontitisD. Catarrhal gingivitisE. I degree parodontosis

94. A 13-year-old patient complainsabout gingival haemorrhage during toothbrushing. Objectively: gums around all theteeth are hyperemic and edematic, PMAindex (papillary marginal alveolary index)is 46%, Greene-Vermillion hygiene indexis 2,5. Provisional diagnosis: exacerbationof chronic generalized catarrhal gingivitis.This patient should be recommended touse a toothpaste with the following activecomponent:

A. ChlorhexidineB. Calcium glycerophosphateC. MonofluorophosphateD. Vitamins A, D, EE. Microelement complex

95. A 30-year-old patient has got anedema and hyperemia of nose wing skinspreading to the skin of his upper lip andcheek. Examination revealed a fissurealong the infra-external edge of the leftnostril. The skin is tense, dense, red-and-cyanotic, skin line pattern is smoothed;rise of local temperature is present. Thezone of affection is well-defined andirregularly shaped. What is the most li-kely diagnosis?

A. Erysipelatous inflammationB. Dermal actinomycosisC. Dermal tuberculosisD. Dermal cancerE. Thrombophlebitis of the facial vein

96. On the second day after tooth extracti-on a 35-year-old woman applied to a facialsurgeon and complained about pain in theparotid-masticatory region and sensationof lacking contact between the upper andlower jaw teeth on the right. Examinationrevealed chin deviation to the left, half-open mouth, closed lips, disturbed occlusi-on. There is also limitation of lateralmandible movements. In front of the rightear tragus soft tissues sink down. Whatexamination is expected to be the mostinformative in this case?

A. TMJ roentgenographyB. TMJ ultrasoundC. Mandible roentgenographyD. Bimanual TMJ investigationE. -

97. A 50-year-old patient has a defect of

the lower dental arch. It is planned tomake an implant-supported bridge for itsrestoration. X-ray picture shows that theheight of the bone mass from projection ofmandibular canal up to the top of alveolarcrest is 2 cm. What type of implant shouldbe applied?

A. ThreadedB. Endodontic-endoosseousC. Plate-formD. SubperiostealE. Conical

98. A 10-year-old child undergoes sanitati-on of the oral cavity. The girl was foundto have chalky spots on the vestibularsurfaces in the precervical region of the21 and 12 teeth. Enamel surface is dull,smooth. Pain reaction to the temperaturestimuli is absent. What additional methodof examination is expected to confirm thediagnosis?

A. Vital stainingB. OrthopantomographyC. Intraoral roentgenographyD. ElectroodontodiagnosticsE. Ultraviolet stomatoscopy

99. A 20-year-old patient complains aboutpain and haemorrhages in the regionof the 36 tooth occuring during eatingsolid food. Objectively: medial masti-catory surface of the 36 tooth has a largecarious cavity occupied by a carneoustumour-like formation, probing induceshaemorrhage and pain in the regionof connection of the carious cavity wi-th the pulp chamber. Percussion is pai-nless. Electroodontodiagnosis is 40 mi-croampere. Roentgenological changes areabsent. What is the most likely diagnosis?

A. Chronic hypertrophic pulpitisB. EpulisC. Hypertrophic papillitisD. Chronic gangrenous pulpitisE. Chronic fibrous pulpitis

100. An infant was born full-term withbody weight at a rate of 3200 g and bodylength at a rate of 53 cm. It was the firstphysiological delivery. What position ofchild’s mandible is usually observed afterbirth?

A. Physiological retrogeniaB. Physiological progeniaC. Direct relationD. Deep overbiteE. Open bite

Page 15: Krok 2 - 2009 Question Paper (Stomatology)

Krok 2 Stomatology 2009 15

101. A 14-year-old child has orthodonticindication for extraction of the 14 tooth.What forceps should be applied forextraction of the 14 tooth?

A. S-shaped forcepsB. Straight forcepsC. Bayonet-shaped forcepsD. Beak-shaped forcepsE. Curved on flat forceps

102. A 14-year-old child was undergoingextraction of the 16 tooth on account ofchronic periodontitis. During the toothextraction it came to perforation of maxi-llary sinus along with penetration of thedistal buccal root into the maxillary sinus.What is the further dentist’s tactics?

A. The patient should be directed to thehospital for a surgical procedureB. The dentist himself should try to extractthe rootC. The dentist should form a clottagewithout informing the patientD. The dentist should close the perforationwith mucoperiosteal graftE. The dentist should perform maxillarysinusotomy in the outpatient setting

103. A 2-month-old child is anxious, sleepsbadly, refuses food, has subfebrile bodytemperature. Objectively: hyperaemicmucous membrane of the child’s tongue,lips, cheeks and palate has a caseous coati-ng that can be easily removed with atampon. Regional lymph nodes are sli-ghtly enlarged and painful on palpation.What disease are these symptoms typicalfor?

A. Acute pseudomembranous candidousstomatitisB. Chronic atrophic candidous stomatitisC. Acute herpetic stomatitisD. DiphtheriaE. Measles-associated stomatitis

104. A 35-year-old patient complainsabout progressing throbbing pain in the26 tooth. Objectively: the 26 tooth has acarious cavity filled with softened denti-ne, tooth cavity is closed, probing of thecavity floor is painless, percussion causesacute pain. There is I degree tooth mobi-lity. Roentgenological changes are absent.What is the most likely diagnosis?

A. Acute purulent periodontitisB. Acute purulent pulpitisC. Acute serous periodontitisD. Exacerbation of chronic periodontitisE. Acute diffuse pulpitis

105. A patient complains of burning,itch and lower lip enlargement. He hasbeen suffering from this for a long ti-me. Objectively: the patient’s face isasymmetric due to the flattening ofnasolabial fold. His lower lip is edematic,of normal colour, painless on palpation.The patient has plicated tongue. What isyour provisional diagnosis?

A. Melkersson-Rosenthal syndromeB. Quincke’s edemaC. LymphangiomaD. HemangiomaE. Granulomatous Miescher’s cheilitis

106. A 30-year-old patient consulted adentist about a cosmetic defect and slightpain in the 44 tooth during tooth brushi-ng. Objectively: there is a Black’s classV carious cavity within mantle dentine.Probing of the cavity walls causes slightpain, thermal probe is positive. What filli-ng material is to be chosen for restorationof this cavity?

A. Compomer restorative materialB. Chemical-cure compositeC. AmalgamD. Polycarboxylate cementE. Metal ceramics

107. An 18-year-old patient complains ofshort-term pain in the 37 tooth causedby sweet and cold stimuli. Objectively:masticatory surface of the 37 tooth hasa carious cavity within mantle dentine.The cavity is filled with softened dentine.Probing of the cavity walls induces painin the region of dentinoenamel junction,electroodontodiagnosis is 6 microampere.What is the most likely diagnosis?

A. Acute median cariesB. Acute superficial cariesC. Acute deep cariesD. Chronic fibrous pulpitisE. Pulp hyperaemia

108. A 24-year-old soldier was injured bya shell splinter. He was diagnosed with amandibular fracture with a bone defect inthe mental region over 2 cm long. Whatmethod of fixation of mandible fragmentsis indicated?

Page 16: Krok 2 - 2009 Question Paper (Stomatology)

Krok 2 Stomatology 2009 16

A. Fragments fixation by means of Rudko’sapparatusB. Tigerstedt’s splintsC. Intermandibular Ivy ligatureD. Gunning-Port’s splintE. Direct osteosynthesis

109. A 35-year-old female patientconsulted a dentist about a painless,slowly growing neoplasm in the area ofthe 11 and 12 teeth. Examination revealedthat the tumour was light-pink, flattened,adjacent to the teeth, had a pedicle. Thetumour was up to 1,5 cm large, withsmooth surface and dense consistency. Itwas diagnosed as an epulis in the regiob ofthe 11 and 12 teeth. What form of epulisare these clinical findings typical for?

A. FibrousB. AngiomatousC. Giant-cellD. Pregnancy epulisE. -

110. A 49-year-old patient applied to theoral surgery department and complainedabout permanent intense dull pain in theregion of the right upper jaw. It is knownfrom the anamnesis that the 17 tooth hasbeen repeatedly treated for exacerbationof chronic periodontitis but the treatmentappeared to be ineffective. What kind ofanesthesia should be applied for extracti-on of the 17 tooth?

A. Tuberal and palatinalB. Tuberal and incisorC. Tuberal, incisor and palatinalD. Incisor and paltinalE. Torus

111. Stomatological examination of a chi-ld revealed abnormal form of the centralincisors: they are barrel-shaped, thereis a semilunar groove on the cuttingedge. It is known from the anamnesisthat the child’s mother had syphilisduring pregnancy. Besides the dentalabnormality the child presents also withdeafness and parenchymatous keratitis.This abnormality of tooth development iscalled:

A. Hutchinson’s teethB. Pfluger teethC. Wedge-shaped defectD. Fluorosis (destructive form)E. Erosion of hard tissues

112. A patient ordered partial removablelamellar dentures for the upper and lowerjaw. An orthodontist made elastic alginate

impressions of both jaws. What is his nextstep?

A. To send the impressions for disinfectionB. To let the impressions dry out in theopen airC. To invite a dental mechanic for jointanalysis of the impressionsD. To send the impressions immediately tothe laboratoryE. To put the impressions into the microtenbag for 90 minutes

113. A 12-year-old patient complainsabout an aesthetic defect. Objectively: thelower third of face is shortened, upperfrontal teeth overbite the lower teethby 3/3 of height, exhibit oral inclinati-on, lateral parts all along exhibit cusp-to-cusp relationship between the antagoni-sts; Angle’s class II malocclusion (joiningof the upper permanent molars) is alsopresent. Malocclusion is observed in thefollowing planes:

A. In sagittal and verticalB. In transversalC. In transversal and verticalD. In verticalE. In sagittal

114. A 65-year-old patient consulted aprosthodontist about fabrication of anexternal prosthesis of orbit that was lostas a result of a trauma. What is the fixingelement of the orbit prosthesis?

A. Spectacle frameB. Watch springC. Swivel devicesD. ClampsE. Magnets

115. A 56-year-old patient needs a partialremovable lamellar denture. Objectively:the 17, 16, 15, 14, 25, 26, 27, 28 teeth on theupper jaw are missing. In order to provi-de transversal line of clasps the clasp armsshould be placed upon the following teeth:

A. 13 and 24B. 13 and 18C. 24 and 18D. 24, 13 and 18E. -

116. A 56-year-old patient consulted aprosthodontist about pain underneath thebar of her clasp prosthesis. It is knownfrom the anamnesis that the clasp wasplaced in another city a week ago. Exami-nation revealed a mechanic injury ofmucous membrane of the palatine vault

Page 17: Krok 2 - 2009 Question Paper (Stomatology)

Krok 2 Stomatology 2009 17

caused by the bar of the clasp prosthesis.What distance should exist between thepalate and the bar of the clasp prosthesisin order to prevent this complication?

A. 0,5 mmB. 2-3 mmC. 0,2-0,3 mmD. 1,5-2,0 mmE. 5-6 mm

117. A 47-year-old patient complainsabout limited mobility of her lower jawin the morning; periodical dull pain inthe right temporomandibular joint (TMJ)and general joint stiffness. Accordingto the patient, the stiffness disappearsthroughout the day after joint "exerci-sing". Objectively: the patient’s face issymmetric, mouth opening is limiteddown to 2,5 cm, there is also joint clicking.Median line deviates to the right by 3-4mm, palpation of the right articular headis painless. What is your provisional di-agnosis?

A. Arthrosis of the right TMJB. Acute serous arthritis of the right TMJC. Chronic arthritis of the right TMJD. Fracure of the right condyle of mandi-bleE. Right-sided anterior dislocation ofmandible

118. A 50-year-old patient complainsabout problems with mastication, toothmobility, halitosis, gingival haemorrhages.Objectively: gums are hyperemic wi-th cyanotic colouring, there is dentalcalculus. Parodontal pouches of thesuperior molars are 8 mm deep, thepouches of other teeth are 6 mm deep.X-ray picture shows resorption of bone ti-ssue by 2/3-1/2 of root length. What is themost likely diagnosis?

A. Chronic generalized periodontitis of IIIdegreeB. Chronic generalized periodontitis of IIdegreeC. Chronic generalized periodontitis of IdegreeD. Acute generalized periodontitis of IIIdegreeE. Acute generalized periodontitis of IIdegree

119. An 18-year-old patient complainsabout gingival enlargement, pain andhaemorrhage when eating solid food.Objectively: hyperaemia, gingival edema,hypertrophy of gingival edge up to 1/2

of crown height by the 12, 13, 14 teeth.Formalin test is painless. What is the mostlikely diagnosis?

A. Hypertrophic gingivitisB. Generalized II degree periodontitis,chronic courseC. Catarrhal gingivitisD. Ulcero-necrotic gingivitisE. Exacerbation of generalized I degreeperiodontitis

120. A 34-year-old patient got a trauma(fall) that resulted in mobility of alveolarprocess and all the upper jaw teeth,occlusion was also changed. X-ray pi-cture depicts the fracture line that runs inboth directions from the piriform openingalong the floor of maxillary sinus. What isthe most likely diagnosis?

A. Le Fort I maxillary fractureB. Partial fracture of the alveolar processC. Le Fort II maxillary fractureD. Le Fort III maxillary fractureE. Unilateral maxillary fracture

121. A boy is 1 month old. At the medi-al edge of the inferior eyelid on the ri-ght there is a wound with purulent di-scharge. The boy fell ill suddenly, bodytemperature rose up to 40oC. The generalcondition is grave. On the second dayof disease there appeared an infiltrati-on at the internal edge of eye socketand right cheek. The skin above it ishyperemic, fluctuation cannot be determi-ned. Palpebral fissure is narrowed. Theright nasal meatus discharges pus. Thereis an infiltration on the vestibular surfaceof alveolar process and on the right palate.Mucous membrane above it is hyperemicalong the mucogingival fold, fluctuationcan be determined. What is the most li-kely diagnosis?

A. Acute hematogenous osteomyelitisB. Acute dacryocystitisC. Phlegmon of the right eye socketD. Acute right-sided highmoritisE. Acute serous periostitis

122. A 20-year-old patient got an injury.Objectively: the patient’s chin and lowerjaw up to the 34 and 45 teeth are missing.The 45, 46, 47, 48, 34, 35, 36, 37 teeth arestable. At what stage of medical evacuati-on the patient will get special medical aid?

Page 18: Krok 2 - 2009 Question Paper (Stomatology)

Krok 2 Stomatology 2009 18

A. Specialized army surgical hospitalB. Battalion aid stationC. Regimental aid stationD. Separate medical detachmentE. Separate medical battalion

123. A 45-year-old patient complainsabout a rapidly growing formation on hislower lip. Examination of the red borderof lips revealed a greyish-red nodule wi-th a hollow in the centre which is filledwith corneous masses that can be easilyremoved. The nodule is painless, mobile.What is your provisional diagnosis?

A. KeratoacanthomaB. PapillomaC. Nodulous verrucous precancer of redborderD. Basal cell carcinomaE. Localized precancerous hyperkeratosisof red border

124. A 60-year-old patient underwentssanitation of the oral cavity before anoperation on account of cataract. Afterexamination the patient was diagnosedwith chronic median caries of the 22 tooth(Black’s class V). What filling materialshould not be used in this patient?

A. Light-cure microhybrid materialB. Chemical-cure microhybrid materialC. Chemical-cure glass ionomer cementD. Silicate cementE. Chemical-cure macrofilled composite

125. A 23-year-old patient complainsabout periodical pain in the region ofthe 11 tooth, protrusion of the alveolarprocess. The patient got a trauma 4 yearsago. Objectively: crown of the 11 toothis dark, percussion is painless. X-ray pi-cture shows roundish well-defined area ofbone tissue rarefication by the root apexof the 11 tooth. The area is 2,0 cm in di-ameter. Puncture results: yellow fluid withcholesterol crystals. What is the most li-kely diagnosis?

A. Maxillary radicular cystB. Chronic maxillary osteomyelitisC. Maxillary ameloblastomaD. Soft maxillary odontomaE. Maxillary osteoclastoma

126. A 45-year-old female patientconsulted a prosthodontist about dentalprosthetics. She works as TV announcer.Objectively: the lower jaw dentition is wi-thout spaces, the upper jaw has a free-end edentulous space and a boundaryedentulous space in the lateral parts. The

remaining 18, 17, 13, 12, 11, 21, 22, 23,24 teeth are stable. What fixing elementsshould be used in a clasp denture forcosmetic purposes?

A. Attachments and bar systemB. Telescopic crownsC. Ney claspsD. Jackson claspsE. Dentoalveolar clasps

127. A 34-year-old patient consulted aprosthodontist about pain and clicking inthe final stage of mouth opening. The pati-ent has a history of a dislocation. Ampli-tude of maximal mouth opening is 58 mm.Mouth opening should be limited down tothe following amplitude:

A. Up to 40-50 mmB. Up to 25-30 mmC. Up to 50-60 mmD. Up to 10-15 mmE. Up to 90-100 mm

128. A 12-year-old patient presents wi-th abnormal position of the upper jawcanine. The 13 tooth is in the vestibularposition, above the occlusal plane. Spacebetween the 14 and the 12 tooth is 6,5 mm.Choose a rational treatment method:

A. InstrumentalB. Surgical and instrumentalC. Surgical and physiotherapeuticD. Instrumental and myogymnasticsE. Surgical and myogymnastics

129. Unused stomatological instrumentswere left on a sterile table at the end ofthe working day. What measures shouldbe taken in order to provide sterility ofthese instruments?

A. Sterilization without preliminaryprocessingB. Disinfection, sterilizationC. Disinfection, presterilization treatment,sterilizationD. Presterilization treatment, sterilizationE. Disinfection only

130. A 42-year-old patient complainsabout gingival pain, progressing gingi-val haemorrhage, increasing tooth mobi-lity, halitosis. Objectively: gums are evi-dently hyperaemic, extremely edematic,they bleed easily on palpation. Tooth rootsare exposed, parodontal pouches are 4-6mm deep, and contain purulent exudate,there is also supragingival and subgingivaldental calculus. II-III grade tooth mobi-lity is present. Orthopantomogram shows

Page 19: Krok 2 - 2009 Question Paper (Stomatology)

Krok 2 Stomatology 2009 19

resorption of interdental septa down to1/2 of their height. What is the most likelydiagnosis?

A. Exacerbation of generalized II degreeperiodontitisB. Exacerbation of generalized I degreeperiodontitisC. Exacerbation of generalized III degreeperiodontitisD. Chronic generalized II degree peri-odontitisE. Chronic generalized III degree peri-odontitis

131. A 60-year-old patient complainsabout difficult mastication due to themandible displacement that resulted froman untreated fracture. Objectively: the35, 36, 38, 45, 46 teeth are missing. Theremaining teeth are intact. The 43, 44, 47,48 teeth have no contact with the upperlateral teeth, and exhibit oral deviation upto 1 cm. What is the optimal prostheticconstruction for the manible?

A. Prosthesis with double dentitionB. Whole-piece bridgeC. Arch bar prosthesisD. Adhesive prosthesisE. Vankevich splint

132. A patient complains about a cosmeticdefect of the 23 tooth. Objectively: thecrown of the 23 tooth is decayed by 80%,the root is stable, the canal is filled up tothe top. After examination it was decidedto restore the decayed tooth with a caststump inlay. The root canal of the 23 toothshould be broadened by:

A. 2/3 of canal lengthB. 1/2 of canal lengthC. 1/3 of canal lengthD. 1/4 of canal lengthE. By the total canal length

133. Regimental Aid Station (RAS) admi-tted a soldier with an injury in themaxillofacial region. Dosimetric controlrevealed radiation affection in this pati-ent. The injured should be referred to thefollowing functional unit of RAS:

A. Decontamination stationB. Dressing pavilionC. Evacuation pavilionD. IsolatorE. Admission and sorting pavilion

134. A 46-year-old patient complai-ns about pain in the left parotid-masticatory region, disorder of mandi-

ble movements. Objectively: skin overthe left temporomandibular joint (TMJ)is reddened, the surrounding tissuesare edematic. Disfunction of the lefttemporomandibular joint is present. Whatis the most likely diagnosis?

A. Acute purulent left-sided arthritis ofTMJB. Acute serous left-sided arthritis of TMJC. Arthrosis of the left TMJD. Deforming arthrosis of the left TMJE. Ankylosis of the left TMJ

135. A patient complains about li-mited mouth opening. She has a hi-story of intra-articular disorders inthe left temporomandibular joint.Roentgenological examination revealedsubchondral sclerosis of articular plates,regular narrowing of articular cavity, limi-ted excursion of condylar process of theleft temporomandibular joint. What is themost likely diagnosis?

A. Arthrosis of the left temporomandi-bular jointB. Deforming arthrosis of the lefttemporomandibular jointC. Ankylosis of the left temporomandi-bular jointD. Acute purulent left-sided arthritis ofthe temporomandibular jointE. Acute serous left-sided arthritis of thetemporomandibular joint

136. A 46-year-old patient complainsof difficult opening of her mouth, bodytemperature rise, tissue edema aroundboth temporomandibular joints (TMJ).It is known from the anamnesis that thepatient had short-term non-intense bi-lateral pain in the parotid-masticatoryregion, limited mouth opening, tensionand discomfort in the region of bothtemporomandibular joints that had beenobserved for a couple of days. What is themost likely diagnosis?

A. Rheumatic arthritisB. Infectional arthritisC. ArthrosisD. Fibrous ankylosisE. Deforming atrhrosis

137. A boy is 10 years old. His face issymmetric and proportional. He presentswith mouth breath. Examination of theoral cavity revealed saddle-like form ofdental arches and high arched palate.Upper first molar relationship (Angle’skey to occlusion) remains intact. What is

Page 20: Krok 2 - 2009 Question Paper (Stomatology)

Krok 2 Stomatology 2009 20

the most likely diagnosis?

A. Narrowing of dental archesB. Distal occlusionC. Mesial occlusionD. Widening of dental archesE. Elongation of dental arches

138. A 56-year-old patient complainsabout a painless neoplasm on the alveolarcrest of the right upper jaw. Objectively:there is a bright-red wide-based tumourin the region of premolars. It is locali-zed on both sides from the alveolar crest.The tumour is dense and elastic, painless.What is the most likely diagnosis?

A. Peripheral osteoclastomaB. Central osteoclastomaC. Gingival fibromatosisD. Fibrous epulisE. Papilloma

139. A 37-year-old patient was admitted tothe oral surgery department. The womancomplains of pain in her cheek that ismade worse by touch. It is known from theanamnesis that five days ago the patientfell down from the stairs and "hurt"hercheek. Objectively: in the depth of cheekexamination revealed a circumscribedinfiltration, cheek skin is hyperaemic anddoesn’t make a fold, fluctuation symptomis present. Mucous membrane is edematicand has teeth indentations. What is themost likely diagnosis?

A. Suppurated cheek haematomaB. Traumatic osteomyelitis of mandibleC. Cheek haematomaD. Cheek phlegmonE. Acute lymphadenitis

140. A 40-year-old patient suffersfrom chronic generalized periodontitis.Objectively: both dentitions are intact,the 42, 41, 31, 32 teeth exhibit I degreemobility. What splint would be aestheti-cally acceptable for this group of teeth?

A. Glass Span and photopolymer splintingB. Guard splintC. Splint of soldered combined crownsD. Cap splintE. Ring splint

141. Renovation of a dental room involvesinstallation of luminous tube lamps. Speci-fy the required illuminance level in lux:

A. 500 luxB. 400 luxC. 300 luxD. 200 luxE. 100 lux

142. A patient complains about a cosmeticdefect. Examination revealed that the 21tooth was missing, the crowns of the 11and the 22 tooth are intact, high, relati-vely parallel. The patient refused teethpreparation as well as implantation orfabrication of a removable denture. Whatorthopedic construction should be fabri-cated?

A. Adhesive dentureB. Clasp dentureC. Lamellar dentureD. Immediate dentureE. Polypropylene denture

143. A 47-year-old patient consulted adoctor about an itching tumour in theright buccal area that has significantlyenlarged lately. Examination revealed anirregularly shaped tumour up to 1,5 cmlarge, the tumour is black and glossy. Whatis your provisional diagnosis?

A. Dermal melanomaB. Pigmented nevusC. Dermal basaliomaD. Dermal cancerE. Cutaneous horn

144. A 38-year-old patient consulted anorthopedist about metal taste, dry mouthand tongue burning. Objectively: defectsof the lower dental arch were replaced wi-th soldered stainless steel bridges. Whatexamination method would be the mostappropriate in this case?

A. GalvanometryB. MasticatiographyC. OcclusiographyD. MyographyE. Electroodontometry

145. A 5-year-old child complains aboutspontaneous pain in an upper jaw tooth onthe right that is getting worse at night andduring eating cold food. Objectively: the65 tooth has a deep cavity communicatingwith the tooth cavity. Probing is painful,percussion is painless. Cold water causeslong-standing pain. What is your provisi-onal diagnosis?

Page 21: Krok 2 - 2009 Question Paper (Stomatology)

Krok 2 Stomatology 2009 21

A. Exacerbation of chronic pulpitisB. Acute periodontitisC. Exacerbation of chronic periodontitisD. Acute serous pulpitisE. Acute purulent pulpitis

146. A 28-year-old patient complainsabout a cosmetic defect in the frontal partof his upper jaw. Objectively: the crownpart of the 11 tooth is decayed below thegum level. The root is stable, percussionis painless. It is planned to restore thetooth with a stump inlay and cover it witha metal-ceramic crown. What additionalmethod of diagnostics should be appliedin this clinical situation?

A. X-ray diagnosticsB. ElectromyographyC. MasticatiographyD. GnathodynamometryE. Electroodontodiagnostics

147. Parents of a 6-year-old child complainabout pain in the child’s submandibularregion on the left, body temperature ri-se up to 37, 5oC. Objectively: the child’sface is asymmetric due to the infiltrationof the submandibular region on the left.The infiltration is soft and elastic, mobile,2х2,5 cm large; its palpation is slightly pai-nful, the skin is unchanged. The teeth areintact. Pharynx is hyperaemic. What is themost likely diagnosis?

A. Acute serous nonodontogenicsubmandibular lymphadenitisB. Acute serous odontogenic submandi-bular lymphadenitisC. Acute purulent nonodontogenicsubmandibular lymphadenitisD. Acute purulent odontogenic submandi-bular lymphadenitisE. Submandibular adenophlegmon

148. A patient complains aboutparoxysmal upper jaw toothache onthe left that is getting worse at night.Toothache intensifies also under stimulati-on and irradiates to the left eye andtemple. Similar attacks were noted threemonths ago, the patient didn’t undergoany treatment. Objectively: the 25 toothhas a deep carious cavity communicati-ng with the tooth cavity. Probing causesacute pain at the point of communicati-on, vertical percussion is slightly pai-nful, horizontal one is painless. Mucousmembrane in the projection of root apexof the 25 tooth is unchanged, its palpati-on is painless. Thermal probe causesacute pain, the pain attack is long-

lasting. Electroodontodiagnosis is 60 mi-croampere. X-ray picture shows slight wi-dening of periodontal fissure at the rootapex of the 25 tooth. What is the mostprobable diagnosis?

A. Exacerbation of chronic pulpitisB. Acute generalized pulpitisC. Acute purulent pulpitisD. Acute purulent periodontitisE. Exacerbation of chronic periodontitis

149. A 23-year-old patient complainsabout gingival haemorrhage during toothbrushing, intensive formation of dentalplaque despite thorough dental care.Objectively: gingival papillae are slightlyedematic, congestively hyperemic, bleedwhen touched. Hygiene index accordingto Fedorov and Volodkina is 3,5. Whattoothpaste would you recommend thispatient as a part of complex therapy?

A. Salt-containing toothpasteB. Toothpaste containing mineralizingcomponentsC. Fluorine-containing toothpasteD. Gel toothpaste with microelementsE. Toothpaste with antifungal agents

150. A pregnant 24-year-old womancomplains about emergence of severalnew carious cavities, falling out of oldfillings. Objective examination revealed:index of decayed, missing and filled teeth(DMF) = 16, Feodorov-Volodkina hygi-ene index is 3,3. Choose the optimalmaterial for carious cavity filling in thiscase:

A. Glass ionomer cementB. Silver amalgamC. Chemical-cure compositeD. Light-cure compositeE. Silicophosphate cement

151. A group of specialists conducts anepidemiological survey of certain agegroups of population aimed at evaluati-on of periodontal disease prevalence andtreatment needs. These rates are studiedby means of the following index:

A. CPITN (WHO index)B. OHI-S (Green-Vermillion index)C. PDI (Ramfjord index)D. PI (Russel index)E. PMA (Parma)

152. After the unproblematic extraction ofthe 37 tooth a 60-year-old patient presentswith profuse haemorrhage from the tooth

Page 22: Krok 2 - 2009 Question Paper (Stomatology)

Krok 2 Stomatology 2009 22

socket. The patient has a 6-year historyof essential hypertension. Now his APis 180/110 mm Hg. What emergency aidshould be rendered?

A. Injection of hypotensive drugs and tightpack of the tooth socketB. Pack of the tooth socket withhaemostatic spongeC. Pack of the tooth socket with iodoformtamponD. Suture ligature of the tooth socketE. Injection of haemostatic drugs

153. A 48-year-old patient complainsabout permanent pain in the region ofthe 38 tooth. She has been suffering fromthis for 3 days. Crown of the 28 tooth iscompletely decayed. What forceps shouldbe applied for extraction of roots of the 28tooth?

A. Bayonet-shaped forcepsB. S-shaped forceps curved leftC. Straight root forcepsD. S-shaped close-beak forcepsE. S-shaped broad-beak forceps

154. During military operations the headof the Army Medical Department gavethe order to deploy a stomatological aidunit on the territory of the Army Hospi-tal Base. A patient with missile wound offace was admitted to the unit. What kindof aid can be rendered in the stomatologi-cal unit?

A. Specialized (secondary) aidB. Qualified aidC. Consultation aidD. Stomatological aidE. Prosthodontic aid

155. A 62-year-old patient has a medianfracture of mandible along with formati-on of a false joint. Objectively: dentalformula is 33, 34, 35, 36, 27, 47, 46, 45, 44,43. The teeth are intact, stable, with highcrowns. Fragment mobility is insignificant,there is no displacement. X-ray pictureshows a bone defect 0,8 cm large. Whatprosthesis is indicated?

A. Bridge-like prosthesis with a pivot pointB. Lamellar prosthesis without a pivotpointC. Lamellar prosthesis with Gavrilow’spivot pointD. Lamellar prosthesis with Oxman’s pivotpointE. Lamellar prosthesis with Weinstein’spivot point

156. A 68-year-old patient complainsabout pain in the palate that occurs duri-ng wearing a complete removable denturethe patient got 3 months ago. Objectively:palatine torus is strongly marked, mucousmembrane around it is hyperaemic andedematic. What is the most probablecause of this complication?

A. Torus wasn’t isolatedB. Previous impression was made in plasterC. Functional impression was taken bymeans of a rigid individual trayD. It was made anatomic teetharrangementE. Prosthesis base is made of acrylic plastic

157. A 27-year-old patient complains of aswelling in the region of her lower jaw onthe right. Objectively: the patient’s face isslightly asymmetric due to thickening ofmandibular body on the right. Adjacentsoft tissues are of unchanged colour andnormal consistency. Mouth opening is notlimited. Mucous membrane of oral cavi-ty presents no changes. In the right lowerdentition the 45 tooth is missing. X-ray pi-cture of the right half of the mandible inits lateral view shows an oval well-defined2x3 cm large radiolucency in the bone ti-ssue. The coronal portion of the retinatedhorizontally positioned 45 tooth is turnedinward the radiolucent area. What is themost probable diagnosis?

A. Follicular cyst of mandibleB. Adamantinoma of mandibleC. Sarcoma of mandibleD. Odontoma of mandibleE. Osteoclastoma of mandible

158. A 52-year-old patient complainsabout a nonhealing ulcer on his lowerlip. The patient is smoker. He hasn’t everconsulted a doctor about it. In the regionof red border of the lower lip a roundi-sh ulcer is present. It is up to 2,0 cm indiameter. The ulcer edges are thickenedand a little bit raised in form of a whitishswelling. In the left submandibular regi-on palpation revealed enlarged, painless,dense lymph nodes with limited mobility.What is the most likely diagnosis?

A. Cancer of the lower lipB. Erosive verrucous leukoplakiaC. KeratoacanthomaD. Fibroma of the lower lipE. Syphilitic ulcer

159. A 25-year-old patient complainsabout acute pain in the mouth, headache,articular pain, body temperature rise up to

Page 23: Krok 2 - 2009 Question Paper (Stomatology)

Krok 2 Stomatology 2009 23

38, 6oC. Red border of lips is covered withhaemorrhagic crusts, mucous membraneof the oral cavity has big erosions andulcers coated with greyish incrustation.Hand skin exhibits erythematous spots 1-1,5 cm in diameter with a vesicle in themiddle. What is the most likely diagnosis?

A. Stevens-Johnson syndromeB. Behcet’s syndromeC. Lyell’s syndromeD. Multiform exudative erythemaE. Medicamentous stomatitis

160. A 14-year-old child complains aboutacute spontaneous spasmodic pain in anupper jaw tooth on the right. The painhas been lasting for 3 days, it is throbbing,irradiating to the temple, getting worse atnight. Objectively: surface of the 15 toothexhibits a carious cavity within parapulpardentine. Dentine is softened, of greyishcolour. Probing of the whole cavity floor ispainful, percussion of the 15 tooth is pai-nless. What is the most likely diagnosis?

A. Acute purulent pulpitisB. Acute diffuse pulpitisC. Acute focal pulpitisD. Acute periodontitisE. Exacerbation of chronic periodontitis

161. A 3-month-old child has an inborntissue defect in the oral cavity. Objecti-vely: the lip is intact, the oral cavity exhi-bits a cleft defect of soft palate and medi-an part of hard palate. The child was di-agnosed with complete schistasis of softpalate and partial schistasis of hard palate.Specify the dispensary group according toA.A.Kolesov:

A. FirstB. SecondC. ThirdD. FourthE. Fith

162. A dentist treats a 22-year-old patienton account of acute deep caries of the 26tooth. As lining material the dentist chosecalcium salicylate cement "Life". Whatmaterial should be chosen for insulationof the lining?

A. Glass ionomer cementB. Insulating varnishC. Adhesive composite systemD. Zinc phosphate cementE. Zinc oxide eugenol cement

163. A 62-year-old patient complains ofa painless formation on his tongue that

appeared several months ago. Objecti-vely: the patient has a lot of cariousand completely decayed teeth, lateralsurface of tongue exhibits a painless whiti-sh formation 10x5 mm large with irregularsurface in form of verrucae. Histologi-cal examination revealed thickening ofcorneal epithelial layer of intermittentkeratinization type. What is the most li-kely diagnosis?

A. Verrucous form of leukoplakiaB. Verrucous precancerC. Hyperplastic form of candidiasisD. Hyperkeratotic form of lichen ruberplanusE. Keratoacanthoma

164. A 35-year-old man has a medi-um deep carious cavity in the 37 tooth(Black’s class II). For its filling a denti-st chose technique of layer-by-layerrestoration. What composite should becoating the floor and walls of the cariouscavity in order to form superadaptive ini-tial layer?

A. FlowableB. CondensableC. MacrofilledD. MicrohybridE. Microfilled

165. A 9-year-old boy presents withface asymmetry due to the chin devi-ation to the left. When the third Il’ina-Marcosian diagnostic test is performedface asymmetry disappears. What is themost likely clinical form of this occlusalanomaly?

A. Habitual deviation of mandibleB. Ankylosis of the temporomandibularjointC. Unilateral hypoplasia of mandibleD. Bilateral narrowing of the maxillarydental archE. Unilateral narrowing of the maxillarydental arch

166. A completely edentulous 70-year-oldpatient has ordered complete removabledentures. Artificial teeth are placed uponthe spherical surface. Close teeth contactby movements of the lower jaw will beguaranteed by the following average radi-us of the spherical surface:

Page 24: Krok 2 - 2009 Question Paper (Stomatology)

Krok 2 Stomatology 2009 24

A. 9 cmB. 5 cmC. 7 cmD. 12 cmE. 18 cm

167. A 40-year-old patient consulted aprosthodontist about upper jaw teethmobility that turned up after therapeutictreatment. Objectively: the 13, 12, 11,21, 22 teeth are pulpless, exhibit IIdegree mobility, unchanged colour. Whatprosthetic construction should be appli-ed?

A. Mamlock splintB. Crown soldered splintC. Elastic crown splintD. Cast bar splintE. Wire-band splint

168. A 5-year-old child was bitten by a dog2 days ago. The child is diagnosed with abite wound of cheek. The parent’s didn’tappeal for medical aid in proper time. Thewound exhibits pyoinflammatory process.What kind of surgical d-bridement shouldbe performed?

A. SecondaryB. Initial earlyC. Initial delayedD. Initial lateE. Surgical procedure is not required

169. A 16-year-old teenager complainsabout halitosis, general weakness, bodytemperature rise up to 37, 6oC. Thesesymptoms turned up 2 days ago, the boyhas a history of recent angina. Objecti-vely: oral cavity hygiene is unsatisfactory,teeth are covered with soft white deposit.Gums are hyperaemic, gingival papillaeare covered with greyish coating. What isthe most likely diagnosis?

A. Ulcero-necrotic gingivitisB. Acute catarrhal gingivitisC. Hypertrohic gingivitisD. Chronic catarrhal gingivitisE. Desquamative gingivitis

170. A patient being at a dentist presentedsuddenly with tachycardia, sweating,trembling, nausea, hunger. The patient isconscious. He suffers from diabetes melli-tus. What aid should be rendered by thedentist?

A. The dentist should break the procedureoff and give the patient sugar in any formB. The dentist should lay the patient downon a flat surface, lower cephalic pole, openthe windowsC. The dentist should give the patientnitroglycerine in form of spray or subli-nguallyD. The dentist should make intramuscularinjection of adrenaline (0,5-1,0 mg)E. The dentist should make intravenousinjection of corticosteroids

171. Tooth replacement with completeremovable dentures involves adjustmentof occlusal relations by differentmovements of the lower jaw. Transversalmovements of the lower jaw are initiatedby the following muscle:

A. External (lateral) pterygoid muscleB. Internal (medial) pterygoid muscleC. Temporal muscleD. Mastication muscleE. Digastric muscle

172. A 50-year-old patient complainsabout pain in the region of the lefttemporomandibular joint (TMJ) duringmouth opening. Dental formula is 14, 13,12, 11, 21, 22, 23, 24, 33, 32, 31, 41, 42, 43,44. While opening the mouth the lowerjaw moves in a zigzag manner towards theaffected joint. What pathological conditi-on are these symptoms typical for?

A. Musculoarticular dysfunctionB. Habitual TMJ dislocationC. Sclerosing arthrosis of TMJD. Chronic arthritis of TMJE. Deforming arthrosis of TMJ

173. A 35-year-old patient is at a denti-st. He is anxious, complains of beingafraid of the dentist, is unwilling to letthe dentist examine him. Drugs of whichgroup should be given the patient beforestomatological manipulations?

A. SedativeB. AnalgeticsC. HypotensiveD. Cardiac glycosidesE. Desensitizing

174. A 76-year-old patient has a roundishulcer in the area of the right nasolabialsulcus. The ulcer has dense, rough floorcovered with yellowish-grey crusts, andresembles of a cone. The ulcer edges aredense, crater-like; the ulcer grows bothsidewards and deepwards. What is yourprovisional diagnosis?

Page 25: Krok 2 - 2009 Question Paper (Stomatology)

Krok 2 Stomatology 2009 25

A. BasaliomaB. Tuberculous ulcerC. Actinomycotic ulcerD. Syphilitic ulcerE. Squamous cell carcinoma of skin

175. A 40-year-old patient complainsabout a long-existing fissure in the centralpart of red border of his lower lip. He hasbeen smoking from the age of 16. Objecti-vely: there is a deep 1,0 cm long fissurealong the median line of red border ofthe lower lip, the fissure edges are bulgi-ngl. Characteristic quality of the fissureis its spontaneous healing, but after epi-thelization the fissure recurs. The pati-ent was diagnosed with chronic fissureof the lower lip. Conservative treatmentproved to be ineffective. Choose the mostappropriate treatment method in this si-tuation:

A. Excision within evidently healthy ti-ssuesB. CryodestructionC. Diathermo-coagulationD. Excision 0,5 cm off the neoplasm limitsE. Close-focus roentgenotherapy

176. It is planned to lance an abscessof hard palate (it is localized close tothe 23, 24, 25 teeth) and to extract thecausative 24 tooth that had caused abscessdevelopment. What type of anaesthesia isindicated for this operation?

A. Infraorbital, palatinal and incisorB. Tuberal, infraorbital and incisorC. Tuberal, infraorbital and palatinalD. Tuberal and incisorE. Tuberal and palatinal

177. A 36-year-old patient consulted adentist about permanent acute pain in theupper jaw teeth, body temperature rise.The dentist found out that the 26 toothhad been repeatedly treated, the rest ofteeth were intact. After roentgenologicalexamination the patient was diagnosedwith acute highmoritis. What is the mosteffective way of treatment?

A. Extraction of the causative tooth andmedicamentous therapyB. Caldwell-Luc maxillary sinusotomyC. Puncture of the maxillary sinus andmedicamentous therapyD. Medicamentous therapyE. Medicamentous and physiotherapeutictreatment

178. A 44-year-old patient complains

about the astringent sensation in theupper jaw incisors, which appeared 1,5years ago. Objectively: the most convexpart of the vestibular surfaces of the 12,11, 21, 22 crowns has roundish defects wi-th smooth, shiny, dense surface, that reachdentine in depth. The depth of defects isgradually decreasing from the centre tothe periphery. What is the most likely di-agnosis?

A. Enamel erosionB. Wedge-shaped defectC. Systemic hypoplasiaD. Chronic median cariesE. Destructive fluorosis

179. Examination of a 6-year-old boyrevealed enlarged lymph nodes in bothsubmandibular and cervical regions.Objectively: the 75, 84 and 85 teethare decayed, there are presentations ofcommissural cheilitis. According to theboy’s mother, he has been quickly getti-ng tired, sweating from the least physi-cal exercise, complaining about weaknessthroughout the last 2-3 months. He alsogave up training in a sports class. Whatplan of additional examination should beadopted?

A. Complete blood count, haematologistconsultationB. Biochemical blood test, endocrinologistconsultationC. Puncture biopsy, oncologist consultationD. Magnetic resonance tomography,immunologist consultationE. Pulmonary roentgenography,pulmonologist consultation

180. A 32-year-old patient complainsabout pain and swelling in the region ofhis mandible and left cheek. He has beensuffering from this for 2 days. Objecti-vely: his general condition is satisfactory.Body temperature is 37, 5oC. There is anedema of soft tissues in the left buccalregion. Submandibular lymph nodes arebilaterally enlarged, painful on palpati-on. Crown of the 37 tooth is partiallydecayed, the tooth is immobile, percussi-on is slightly painful. Mucogingival foldin the region of the 37 tooth is edematic,hyperaemic, flattened, its palpation causesacute pain. What is the most likely di-agnosis?

Page 26: Krok 2 - 2009 Question Paper (Stomatology)

Krok 2 Stomatology 2009 26

A. Acute purulent periostitis of mandiblebeginning from the 37 toothB. Acute odontogenous osteomyelitis ofmandibleC. Chronic productive periostitis of mandi-ble beginning from the 37 toothD. Chronic odontogenous osteomyelitis ofmandibleE. Abscess of alveololingual groove on theleft

181. A 12-year-old girl complains aboutswelling, reddening and itching of lips. Shehas a history of bronchial asthma. Suchcondition has been observed in the childfor 3 years in winter period. Objectively:red border of lips and adjacent skin exhi-bit a well-defined erythema and edema,small cracks. There are signs of lip li-chenification. There are also marks ofscratches on the skin of lips, cheeks andforehead in form of abrasions and crusts.Mucous membrane exhibits no pathologi-cal changes. What is the most likely di-agnosis?

A. Atopic cheilitisB. Contact allergic cheilitisC. Meteorological cheilitisD. Exfoliative cheilitisE. Microbal cheilitis

182. Parents of a 5-year-old child notedirregular colour of the child’s back oftongue and consulted a pedodontistabout this. Objectively: the oral cavityis sanitized, DF index = 4, dorsal andlateral surfaces of tongue have area ofdesquamating epithelium bordering uponareas of hypertrophy and increased corni-fication of filiform papillae. The childhas a history of being allergic to somefoodstuffs. What is the clinical diagnosis?

A. Desquamative glossitisB. Tuberculous glossitisC. Candidous glossitisD. Herpetic tongue lesionE. Acute catarrhal glossitis

183. A 4-year-old girl was bitten by a dogin her upper lip. Which of the followingsolutions should be primarily used for thewound d-bridement?

A. 10% solution of laundry soapB. 0,002% solution of chlorhexidineC. 1% solution of hydrogen oxideD. 1 : 5000 solution of potassiumpermanganateE. 3% soda solution

184. A 12-year-old boy complains about

permanent intense throbbing toothachethat is getting worse when biting downon food. Objectively: the patient’s faceis asymmetric because of a collateraledema of soft tissues, submandibularlymph nodes are enlarged and painful onpalpation. The 26 tooth has a deep cavitynot communicating with the tooth cavity.Thermal test is positive, probing is pai-nless, percussion causes acute pain, thetooth is mobile. X-ray picture of the 26tooth shows no changes in the periodonti-um. What is the most likely diagnosis?

A. Acute purulent periodontitisB. Acute serous periodontitisC. Exacerbation of chronic periodontitisD. Acute purulent pulpitisE. Pulpitis complicated by periodontitis

185. A 6-year-old girl was brought to thedentist for completing the treatment ofthe 75 tooth for chronic granulating peri-odontitis. The patient has no complaints.Objectively: occlusive dressing on the 75tooth remains intact, percussion is pai-nless, mucous membrane in the regionof the 75 tooth exhibits no pathologicalchanges, is painless on palpation. Whatmaterial is the most appropriate for theroot canal filling in this case?

A. Zinc oxide eugenol pasteB. Sealer with gutta-percha pointC. Calcium-containing pasteD. Glass ionomer cementE. Iodoform paste

186. A 4-year-old child presents withheadache, nausea, pain when swallowi-ng. Body temperature is 39oC. Examinati-on of the oral cavity revealed extremelyhyperaemic mucous membrane and tonsi-ls. On the first day of disease the child’stongue was furred with greyish coating,on the second day the tongue was foundto be self-purified. On the third day thetongue was smooth, of crimson colour, wi-th hyperaemic fungiform papillae. What ispresumptive diagnosis?

A. Scarlet feverB. VaricellaC. MeaslesD. DiphtheriaE. Herpangina

187. A 15-year-old child presents withpuffiness in the region of the mandi-ble branch; enlarged, dense and painlesslymph nodes adhering to the surroundingtissues. X-ray picture of mandible branchshows a well-defined bone resorption

Page 27: Krok 2 - 2009 Question Paper (Stomatology)

Krok 2 Stomatology 2009 27

area containing small sequestra. AfterMantoux test a 12 mm papule was noted.What is the most likely diagnosis?

A. Tuberculosis of mandible branchB. Mandibular actinomycosisC. Chronic osteomyelitis of mandiblebranchD. Acute mandibular osteomyelitisE. Ewing’s sarcoma

188. An 11-year-old child complains aboutmissing crown of the 12 tooth as a resultof a trauma. The tooth root is well treated.What prosthetic construction is indicatedfor elimination of this defect?

A. Il’ina-Marcosian’s pivot toothB. Cantilever prosthesis supported by the11 toothC. Cantilever prosthesis supported by the13 toothD. Bridge-like prosthesis supported by the13 and 11 teethE. Partial removable replacing prosthesis

189. A 34-year-old patient has indicati-on for extraction of the 38 tooth becauseof chronic fibrous periodontitis. Oralsurgeon performed torus anaesthesia.This anaesthesia blocked transmissionof pain information from the followingnerves:

A. Infraalveolar, lingual, buccalB. Lingual, buccalC. Infraalveolar, buccalD. Infraalveolar, lingualE. Lingual, buccal, mental

190. A patient got a trauma thatresulted in limited mouth opening, nasalhaemorrhage, numbness of inferior eyelidas well as skin in the infraorbital area.Objectively: there is face deformationcaused by depression of soft tissues in themalar region on the left; there is also astep-off deformity in the median part ofthe infraorbital rim and in the region ofzygomatico-alveolar crest. What methodof examination will be the most effectivefor the diagnostics?

A. X-ray study of facial skull in the axialprojectionB. X-ray study of facial skull in the frontalprojectionC. X-ray study of facial skull in the lateralprojectionD. X-ray study of facial skull and paranasalsinusesE. Orthopantomogram

191. A 22-year-old patient is afraid of painfrom conduction anaesthesia. A dentistdecided that this anaesthesia should bepreceded by applicational anaesthesia ofmucous membrane on the spot of injecti-on. What drug should be used for thispurpose?

A. 5% lidocaine ointmentB. 1% synthomycin ointmentC. 5% oxacillin ointmentD. 3% sinaflan ointmentE. 3% doxycyclin ointment

192. A 12-year-old girl complains aboutintense pain in the region of the 46 toothsocket that was extracted 3 days ago. Thepain is irradiating along the branches oftrigeminus. Objectively: lymph nodes areenlarged and painful on palpation, tissuesaround the tooth socket are edematicand hyperaemic. The socket walls arecovered with grey-and-green depositionwith putrid smell. What is the most likelydiagnosis?

A. AlveolitisB. PericoronaritisC. OstitisD. PeriostitisE. Osteomyelitis

193. A 42-year-old patient consulteda dentist about intense lancinatingparoxysmal pain accompanied by asensation of current passage in the regi-on of her upper lip on the right. Painattacks occur spontaneously and last 3-5 minutes. The patient usually has 2-3attacks a day. The patient is unable toestablish the cause of this disease. Exami-nation of her oral cavity revealed nopathological changes. What is the mostlikely diagnosis?

A. Peripheral neuralgia of the II branch oftrigeminusB. Pterygopalatine ganglionitisC. Neuritis of the II branch of trigeminusD. Central neuralgia of the II branch oftrigeminusE. Right-sided upper jaw pulpitis

194. A 23-year-old patient was deli-vered to a traumatology centre witha dirty cut wound of her right foot.A doctor performed initial surgical d-bridement and made an injection of anti-tetanus serum. Some time later the pati-ent’s condition got abruptly worse: shedeveloped extreme weakness, dizziness,palpitation. Objectively: the skin is pale,the patient has cold sweat, frequent pulse

Page 28: Krok 2 - 2009 Question Paper (Stomatology)

Krok 2 Stomatology 2009 28

of poor volume at a rate of 100 bpm, APis 90/40 mm Hg. What is the cause of suchaggravation?

A. Anaphylactic shockB. Haemorrhagic shockC. Pain shockD. Infectious-toxic shockE. Drug disease (seroreaction)

195. A man was found in a streetin unconscious state and delivered tothe sanitary inspection room. At smallintervals the man presents with recurrentattacks of tonic and clonic convulsions ofthe upper and lower limbs. Objectively:the patient is unconscious, his pupils aremydriatic, don’t react to light. The tonguehas teeth indentations. Involuntary uri-nation was noted. Examination revealedno symptoms of focal cerebral lesion. APis 140/90 mm Hg. The patients head canbe freely brought to the chest. Specify themost probable pathology:

A. Epilepsy, status epilepticusB. Parenchymatous hemorrhageC. Cerebral infractionD. TetanusE. Acute meningoencephalitis

196. A patient was found unconsciousin her flat heated with firewood. Thestove pipe damper was closed, it smelt ofsmoke in the room. Objectively: the pati-ent is unconscious, her skin and mucousmembranes are of cherry red colour. Pupi-ls exhibit no reaction to light, corneal andtendinous reflexes are absent, muscle toneis decreased. Ps is 96 bpm, of poor volume.Heart sounds are muffled, there are soli-tary extrasystoles, AP is 90/60 mm Hg.In the lower lungs solitary small bubbli-ng rales are present, respiratory rate is28/min. What is the most likely diagnosis?

A. Acute intoxication with carbon oxideB. Acute chlorine intoxicationC. Acute intoxication with sulfur dioxidegasD. Acute methane intoxicationE. Acute benzol intoxication

197. During transportation of a 60-year-old patient an emergency doctor notedaggravation of his general condition,pulselessness, mydriatic pupils, absence ofrespiratory movements. It will take 5 mi-nutes to get to the admission ward. Whatactions should the doctor take in the firstplace?

A. Start cardiopulmonary resuscitationB. Deliver the patient to the hospital assoon as possibleC. Make an intracardiac injection ofadrenalineD. Facilitate oxygen inhalationE. Make an intravenous injection ofpolyglucinum

198. An 18-year-old woman has beensuffering from diabetes mellitus for 5years. She receives 36 units of insulin perday. During pneumonia her condition gotabruptly worse: the patient presented withincreased thirst, abdominal pain, nausea,vomiting and sleepiness. In the eveningthe patient refused to eat and didn’t gether regular insulin dose; in the morningshe lost consciousness. Objectively: thepatient is unconscious, the skin is dry,turgor is decreased. The tongue is alsodry. Respiration is deep and noisy, there issharp smell of acetone from the mouth.Body temperature is 36, 6oC, Ps is 100bpm, small; AP is 90/50 mm Hg. Urineexhibits extremely positive reaction toacetone. Blood glucose is 33 millimole/l.What is your

A. Ketoacidotic comaB. Hyperosmolar comaC. Lactacidemic comaD. Hepatic comaE. Cerebral coma

199. A 28-year-old man attended aglasshouse in a botanic garden. After hehad smelt at an orchid he turned pale andlost consciousness. Objectively: heart rateis 115/min, arterial pressure is 50/0 mmHg. What drug should be injected to thepatient in the first place?

A. PrednisoloneB. CordiaminC. StrophanthineD. MesatonE. Dimedrol

200. A 68-year-old patient has been takingdiclofenac because of pain in the knee joi-nts for about a year. In the evening aftersubstantial meal the patient presented wi-th liquid black stool, drastic weakness,single vomiting after which the patientlost consciousness. Emergency team deli-vered him to the hospital. Objectively:the patient is conscious, with slightlydeferred reaction, pale. Heart rate is 20breaths/min, Ps is 102 bpm, AP is 110/70mm Hg. Abdomen is soft, painful in theepigastrium. What is the most likely di-

Page 29: Krok 2 - 2009 Question Paper (Stomatology)

Krok 2 Stomatology 2009 29

agnosis? A. Gastrointestinal haemorrhageB. Food poisoningC. Myocardium infarctionD. Apoplectic attackE. Thrombosis of mesenterial arteries